Neoplasm

Pataasin ang iyong marka sa homework at exams ngayon gamit ang Quizwiz!

2011 - 14 In patients with androgen-independent metastatic prostate cancer, the median improvement in overall survival of docetaxel + prednisone every three weeks compared to mitoxantrone + prednisone is: A. 2.5 months. B. 4.5 months. C. 6 months. D. 12 months. E. 18 months.

A . Docetaxel is currently the only FDA-approved agent that has been shown to prolong survival in men with androgen-independent metastatic prostate cancer. In the pivotal trial of docetaxel, patients who received an every three week administration of the drug had a median survival of 18.9 months, as opposed to a 17.3 month median survival for patients who received docetaxel on a weekly basis and 16.4 months (difference of 2.5 months) for those who received mitoxantrone. The p-value comparing docetaxel every three weeks to mitoxantrone (which does not prolong survival but improves quality of life) was p=0.009. These findings led the FDA to approve docetaxel for use in these patients. Tannock I, DeWit R, Berry W, et al: Docetaxel plus prednisone or mitoxantrone plus prednisone for advanced prostate cancer. NEJM 2004;351:1502-1512. Eisenberger MA, Carducci M: Treatment of hormone-refractory prostate cancer, in Wein AJ, Kavoussi LR, Novick AC, Partin AW, Peters CA (eds): CAMPBELL'S UROLOGY, ed 9. Philadelphia, Saunders Elsevier, 2007, vol 3, chap 105, pp 3108-3110.

2014 - 80 A 23-year-old man has a persistent 5 cm interaortocaval mass after three cycles of BEP for left NSGCT. Serum tumor markers are normal. During RPLND, the mass is densely adherent to the aorta and vena cava. An incisional biopsy of the mass reveals fibrosis. The next step is: A. complete RPLND. B. sample para-aortic and paracaval nodes. C. completely resect residual mass. D. terminate RPLND and follow tumor markers. E. terminate RPLND and give XRT.

A . Following chemotherapy for retroperitoneal metastases from non-seminomatous germ cell tumors, teratoma will be found in approximately 40% of patients. When this is found, complete resection is mandatory for a number of reasons. Although the teratoma may be benign, continued growth of the tumor can lead to significant morbidity and the tumor can become unresectable. Additionally, complete retroperitoneal nodal evaluation will detect foci of residual cancer in up to 20% of patients. Thus, complete RPLND is necessary. Sheinfeld J, Bosl GJ: Surgery of testicular tumors, Wein AJ, Kavoussi LR, Novick AC, Partin AW, Peters CA (eds): CAMPBELL-WALSH UROLOGY, ed 10. Philadelphia, Elsevier Saunders, 2012, vol 1, chap 32, p 875.

2013 - 134 A 54-year-old man with metastatic clear cell RCC is currently receiving sunitinib. During therapy, he should have monitoring of his serum: A. thyroid stimulating hormone and T4. B. testosterone. C. cholesterol. D. cortisol. E. transaminases.

A . Hypothyroidism has been reported in 36% to 46% of patients who took sunitinib in prospective studies. A higher incidence (53% to 85%) has been reported in studies containing both retrospective and prospective data. The mean time to onset of hypothyroidism after initiation of sunitinib therapy ranged from 12 to 50 weeks. The risk of development of hypothyroidism appears to increase with the increasing duration of sunitinib therapy, and the condition is likely reversible once therapy has been discontinued. Baseline thyroid function tests should be performed before the initiation of sunitinib treatment. Because hypothyroidism can develop early in the course of therapy, thyroid function tests should be monitored frequently throughout the duration of treatment. Possible mechanisms for thyroid dysfunction include impaired thyroid hormone synthesis, a destructive thyroiditis preceding the development of hypothyroidism, and increased thyroid hormone clearance. If hypothyroidism is identified, levothyroxine therapy should be promptly initiated. Sunitinib does not affect testosterone, cholesterol, cortisol or liver function. Vetter ML, Kaul S, Iqbal N: Tyrosine kinase inhibitors and the thyroid as both an unintended and an intended target. ENDOCR PRACT 2008;14:618-624.

2013 - 98 A 55-year-old woman undergoes right radical nephrectomy and inferior vena cava thrombectomy for RCC. There is no evidence of metastatic disease. The prognostic factor most predictive of cancer-free survival is: A. tumor stage. B. tumor grade. C. size of caval thrombus. D. tumor size. E. mutant p53 suppressor gene.

A . Important prognostic factors for RCC include specific clinical signs or symptoms such as anemia, hematuria and weight loss; tumor-related factors such as grade and histology; and various laboratory findings such as hypercalcemia. Although an integrative approach utilizing nomograms and risk tables combining a variety of factors have proven to be powerful analyses tools, the local tumor stage remains the most important single prognostic factor for RCC. The cephalad extent of tumor thrombus has been associated with outcome, since a thrombus above the diaphragm increases the stage. However, the absolute size of thrombus does not correlate to outcome. Presence or absence of p53 mutation is not prognostic in RCC. Campbell SC, Lane BR: Malignant renal tumors, Wein AJ, Kavoussi LR, Novick AC, Partin AW, Peters CA (eds): CAMPBELL-WALSH UROLOGY, ed 10. Philadelphia, Elsevier Saunders, 2012, vol 2, chap 49, pp 1443-1444. Glazer AA, Novick AC: Long-term followup after surgical treatment for renal cell carcinoma extending into the right atrium. J UROL 1996;155:448. Reissigl A, Janetschek G, Eberle, et al: Renal cell carcinoma extending into the vena cava: Surgical approach, technique and results. BR J UROL 1995;75:138. Swierzewski DJ, Swierzewski MJ, Libertino JA: Radical nephrectomy in patients with renal cell carcinoma with venous, venal caval, and atrial extension. AM J SUR 1994;168:205.

2012 - 140 In a 65-year-old man with RCC, the panel of molecular markers that would be most predictive of good prognosis is: A. high carbonic anhydrase IX, absent vimentin, absent p 53 expression. B. low carbonic anhydrase IX, positive vimentin, absent p53 expression. C. high carbonic anhydrase IX, positive vimentin, positive p53 expression. D. low carbonic anhydrase IX, absent vimentin, positive p53 expression. E. high carbonic anhydrase IX, positive vimentin, positive p53 expression.

A . In multivariate analysis of a microarray of multiple RCC, certain molecular markers were independently predictive of survival in patients. Those that were predictive of longer survival included high carbonic anhydrase IX (CAIX), absent vimentin and absent p53 expression. CAIX is a von Hippel-Lindau mediated enzyme. High expression of CAIX predicts favorable prognosis. These factors were predictive independent of clinical variables. Tununguntla HSGR, Jorda M: Diagnostic and prognostic molecular markers in renal cell carcinoma. J UROL 179:2008:2096-2102.

2014 - 116 An independent factor associated with poor prognosis in patients with local recurrence of RCC following radical nephrectomy is: A. pulmonary metastasis. B. age > 65 years. C. ipsilateral adrenal location. D. time to local recurrence. E. size of local recurrence.

A . Local recurrence after radical nephrectomy is rare with an incidence of 2.9%. In addition, it is associated with a poor prognosis. Patients with local recurrence treated nonsurgically have a low survival rate. In addition, the presence of synchronous metastasis is associated with a significantly lower survival rate. Best outcomes are reported in those patients treated with surgical resection for isolated local recurrence only. Other factors, including time to local recurrence, location in the ipsilateral adrenal or renal fossa, and size of local recurrence are not independently associated with a worse outcome. Overall, five year survival rates for patients with surgically resected isolated local recurrences are approximately 30%. Bruno JJ, Snyder ME, Motzer RJ, et al: Renal cell carcinoma local recurrences: Impact of surgical treatment and concomitant metastasis on survival. UROL ONCOL 2006;97:933-938. Campbell SC, Lane BR: Malignant renal tumors, Wein AJ, Kavoussi LR, Novick AC, Partin AW, Peters CA (eds): CAMPBELL-WALSH UROLOGY, ed 10. Philadelphia, Elsevier Saunders, 2012, vol 2, chap 49, p 1467.

2012 - 80 A 64-year-old man has painless right testicular swelling of three months duration. Urinalysis is normal, and testicular ultrasound reveals an enlarged right testis with multiple hypoechoic lesions. The most likely diagnosis is: A. lymphoma. B. chronic lymphocytic leukemia. C. spermatocytic seminoma. D. teratocarcinoma. E. embryonal cell carcinoma.

A . Lymphoma accounts for about 5% of all testicular tumors. It is the most common of all testis tumors over the age of 50. The median age of occurrence is approximately 60 years. Lymphoma is the most common secondary neoplasm of the testis. Microscopically, all varieties of reticuloendothelial neoplasms, including Hodgkin's disease, have been described in the testis. The vast majority, however, are diffuse; of these, most are histiocytic. Of germ cell tumors in men over 50, seminoma is most common. However, in the presence of multiple testicular lesions in a 64-year-old man, lymphoma is the most likely diagnosis. Richie JP, Steele GS: Neoplasms of the testis, in Wein AJ, Kavoussi LR, Novick AC, Partin AW, Peters CA (eds): CAMPBELL'S UROLOGY, ed 9. Philadelphia, Saunders Elsevier, 2007, vol 1, chap 29, pp 930-931.

2013 - 27 A 25-year-old man has a solid testes mass. His tumor markers are negative. He has an 8 cm retroperitoneal mass and multiple 1-2 cm. pulmonary metastases. His radical orchiectomy reveals pure seminoma. After chemotherapy his retroperitoneal mass is 2.8 cm and his pulmonary masses have resolved. PET/CT reveals no enhancement of his retroperitoneal mass. The next step is: A. observation. B. percutaneous biopsy of retroperitoneal mass. C. resection of retroperitoneal mass. D. bilateral RPLND. E. salvage chemotherapy.

A . PET imaging is useful to assess post-chemotherapy residual masses after treatment of seminoma. Lesions that are less than 3 cm or non-enhancing can be safely observed as over 90% of seminoma postchemotherapy masses are fibrosis. Percutaneous biopsy is not reliable since the masses can be heterogeneous. Resection of non-enhancing masses is not necessary and resection of seminoma post-chemotherapy masses can be technically difficult or impossible. Resection of the mass and bilateral RPLND would be appropriate for NSGCT post-chemo masses but are not necessary in the post-chemo seminoma setting because of the low risk of cancer or teratoma in the remainder of the retroperitoneum. Salvage chemotherapy is not necessary and is highly toxic. Stephenson AJ, Gilligan TD: Neoplasms of the testis, Wein AJ, Kavoussi LR, Novick AC, Partin AW, Peters CA (eds): CAMPBELL-WALSH UROLOGY, ed 10. Philadelphia, Elsevier Saunders, 2012, vol 1, chap 31, p 837.

2012 - 92 The most frequent complications associated with the use of mitomycin C for intravesical therapy are: A. chemical cystitis and rash. B. myelosuppression and rash. C. flu-like symptoms and myelosuppression. D. contracted bladder and chemical cystitis. E. myelosuppression and chemical cystitis.

A . Rash occurs in 9% of patients receiving mitomycin C instillations, and may represent a contact dermatitis. Chemical cystitis has been reported in 6-41% of patients managed with this agent. The molecular weight of mitomycin C is so high that little is absorbed and myelosuppression is rare. A contracted bladder is also rare after mitomycin C treatment. Flu-like symptoms, which are commonly seen after BCG and interferon therapy, are uncommon after intravesical chemotherapy. Taneja SS, Chan S: Complications of intravesical therapy, in Taneja SS, Smith RB, Ehrlich RM (eds): COMPLICATIONS OF UROLOGIC SURGERY. PREVENTION AND MANAGEMENT, ed 3. Philadelphia, WB Saunders Company, 2001, chap 8, p 87. Jones JS, Campbell SC: Non-muscle-invasive bladder cancer (Ta, T1, and CIS), in Wein AJ, Kavoussi LR, Novick AC, Partin AW, Peters CA (eds): CAMPBELL'S UROLOGY, ed 9. Philadelphia, Saunders Elsevier, 2007, vol 3, chap 76, p 2459.

2014 - 119 A 49-year-old man had a hand-assisted laparoscopic radical nephrectomy for a 3.8 cm renal mass. Pathologic analysis reveals a grade 3 clear cell RCC confined to the kidney and three regional lymph nodes are negative. The most appropriate follow-up for this patient in addition to routine history and physical exam is: A. labs and chest x-ray yearly. B. labs, chest x-ray, and CT scan yearly. C. labs, chest x-ray, CT scan, and bone scan yearly. D. labs and chest x-ray every six months for three years, then yearly. E. labs, chest x-ray, and CT scan every six months for three years, then yearly.

A . Recent studies on the outcome of radial nephrectomy for localized RCC have demonstrated that the risk of recurrence is stage dependent with the risk approximately 7%, 25%, and 40% for T1, T2, and T3 N0 M0 patients, respectively. The risk of recurrence is also greatest during the first three years. The AUA Guidelines for follow-up of clinically localized renal neoplasms has recently been published. Evidence based recommendations for follow-up should be tailored according to pathologic stage. This patient has stage T1aNo disease. For patients with stage pT1 disease, routine abdominal CT scans are not necessary, however, a yearly chest x-ray for three years is now recommended per AUA Guidelines. For stage pT2 and pT3 patients, it is recommended to obtain chest x-ray and CT scan every six months for three years, then annually to year five. Donat SM, Chang SS, Bishoff JT, et al: Follow-up for clinically localized renal neoplasms: AUA Guideline. FOLLOW-UP CARE FOR RENAL CANCER. American Urological Association Education and Research, Inc, 2013. http://www.auanet.org/education/guidelines/renal-cancer-follow-up.cfm

2011 - 36 A 74-year-old man has a 2 cm lower pole renal mass that enhances on CT scan. His medical history includes hypertension, congestive heart failure, and renal insufficiency with a creatinine of 1.8 mg/dl. The next step is: A. renal mass biopsy. B. cryoablation. C. radiofrequency ablation. D. partial nephrectomy. E. radical nephrectomy.

A . Renal mass biopsy should now be considered in select patients with small renal masses to help stratify oncologic risk and offer the optimal treatment intervention. Most studies suggest that biopsy has an accuracy of over 90% in distinguishing benign vs. malignant histology and an associated 70-80% accuracy in assessing tumor histology and grade. Needle tract seeding is exceedingly rare. This patient has comorbidities that might encourage surveillance of his small renal mass, but a minimally invasive treatment would also be appropriate for an aggressive histology given the favorable tumor size and location. Knowing the histology could assist in counseling this patient, particularly since ~20% of small renal masses are benign. Radical nephrectomy would be inappropriate in treating a small exophytic lesion in a patient with renal insufficiency. Given the patient comorbidity and the potential morbidity of partial nephrectomy, a less invasive approach is favored in this setting. This patient is also a reasonable candidate for observation. Lane BR, Campbell SC: Management of small renal masses. AUA UPDATE SERIES 2009, vol 28, lesson 34, pp 314-317. Novick AC, Campbell, SC, Belldegrun A, et al: Guideline for management of the clinical stage 1 renal mass. MANAGEMENT OF THE CLINICAL STAGE 1 RENAL MASS GUIDELINE. Urological Association Education and Research, Inc, 2009.

2012 - 24 A 68-year-old man with advanced prostate cancer is to receive sipuleucel-T (PROVENGE«). Premedication should include acetaminophen and a(n): A. antihistamine. B. mineralocorticoid. C. glucocorticoid. D. benzodiazepine. E. opioid.

A . Sipuleucel-T is an active cellular immunotherapy that is a type of therapeutic cancer vaccine. It consists of autologous peripheral blood mononuclear cells with antigen presenting cells that have been activated ex-vivo with a recombinant fusion protein that consists of prostatic acid phosphatase that is fused to granulocyte-macrophage colony-stimulating factor (an immune-cell activator). In men with asymptomatic or minimally symptomatic castrate-resistant prostate cancer, a 4.1 month median overall survival was demonstrated compared to placebo. The most common side effects included chills, fatigue, and pyrexia which are common with release of cytokines. The recommended premedications are acetaminophen and an antihistamine. Glucocorticoids and opioids are sometimes given at the time of I.V. chemotherapy but are not indicated at the time of immune therapy. Mineralocorticoids and anxiolytics are not indicated for this immunotherapy. Kantoff PW, Higano CS, Shore ND, et al: Sipuleucel-T immunotherapy for castration-resistant prostate cancer. NEJM 2010;363:411-422.

2012 - 134 A 55-year-old man seeks consultation regarding prostate cancer screening. According to the U.S. Prostate, Lung, Colon, and Ovarian (PLCO) trial, prostate cancer screening increases: A. prostate cancer detection. B. detection of Gleason's score 8-10 tumors. C. detection of high stage (advanced) tumors. D. prostate cancer survival. E. quality of life in men with elevated PSA.

A . The Prostate, Lung, Colorectal, and Ovarian (PLCO) Cancer Screening Trial randomly assigned 76,693 men to receive either annual screening (38,343 subjects) or usual care as the control (38,350 subjects). Men in the screening group were offered annual PSA testing for six years and DRE for four years. After seven years of follow-up, the incidence of prostate cancer was higher in the screened population; the number of prostate cancers per 10,000 person-years was 116 (2820 cancers) in the screening group and 95 (2322 cancers) in the control group. Prostate cancer survival was similar between study groups; the incidence of death per 10,000 person-years was 2.0 (50 deaths) in the screening group and 1.7 (44 deaths) in the control group. The numbers of subjects with advanced (stage III or IV) tumors were similar in the two groups, with 122 in the screening group and 135 in the control group, though the number of subjects with a Gleason score of 8 to 10 was higher in the control group (341 subjects) than in the screening group (289 subjects). Quality of life was not assessed. Thus, in this US trial, screening did improve detection rate; however, at a median follow-up of seven years, there was no difference in survival between the two groups. Andriole GL, Grubb III RL, Buys SS, et al: Mortality results from a Randomized Prostate-Cancer Screening Trial. NEJM 2009;360:1310-1309.

2013 - 33 A 40-year-old newly-diagnosed HIV positive man has a 2.0 cm. painless red nodule on his glans penis. A biopsy confirms Kaposi's sarcoma. The next step is: A. initiate highly active antiretroviral therapy (HAART). B. systemic chemotherapy. C. laser ablation. D. excise the lesion. E. partial penectomy.

A . The first step in treatment of Kaposi's sarcoma in patients with HIV is to initiate HAART or to optimize the HAART regimen, which generally results in remission of Kaposi's sarcoma. Local treatment can include laser therapy, cryotherapy, surgical excision, application of topical retinoids. Disseminated or visceral Kaposi's sarcoma is treated with combination chemotherapy. The gold standard combination therapy of doxorubicin, bleomycin and vincristine has been replaced in recent years with liposomal anthracyclines, such as doxorubicin. Kaposi's sarcoma is also often seen in immunosuppressed patients, such as renal transplant patients, and in this setting the treatment is a reduction of the immunosuppressive regimen. In the current era of immunosuppression, the frequency of this is diminished. Heyns CF, Groeneveld AE, Sigarroa NB: Urologic complications of HIV and AIDS. NAT CLIN PRACT UROL 2009;6:32-43.

2012 - 125 A 55-year-old woman with metastatic RCC has received five months of therapy with sunitinib. She undergoes uneventful right laparoscopic radical nephrectomy for a 5 cm upper pole renal mass. On post-op day one, she is obtunded, febrile, and complains of nausea and diffuse abdominal pain. BP is 80/50 mm Hg, HR 78 bpm, and urine output is 30 ml over four hours. Laboratory studies show a stable hemoglobin of 13.5 g/dl, and normal WBC. Chemistry studies are pending. The next step is normal saline fluid bolus and: A. I.V. dexamethasone. B. I.V. desmopressin. C. CT scan of abdomen and pelvis. D. angiography. E. surgical exploration.

A . The patient described above has the classic symptoms and signs of acute adrenal insufficiency. The most common symptoms and signs of acute adrenal insufficiency occurring in the setting of clinical deterioration are fever, nausea/vomiting, abdominal or flank pain, hypotension, abdominal distention, lethargy/obtundation, hyponatremia, and hypokalemia. The urologist should have a high index of suspicion for acute adrenal insufficiency following radical nephrectomy and the degree should be heightened in a patient receiving sunitinib as it may contribute to the condition. Physicians are advised to monitor for adrenal insufficiency in patients treated with sunitinib who experience stress such as surgery. As this was an upper-pole mass, the ipsilateral adrenal may have been removed or devascularized as well. Emergency treatment of adrenal crisis is normal saline fluid resuscitation, and dexamethasone sodium phosphate injection (Decadron, 4 mg I.V.). Stat serum electrolytes, glucose, cortisol, and plasma ACTH are obtained. A short ACTH stim test to confirm the diagnosis of adrenal insufficiency is then performed. Supportive measures are provided as needed. Mineralocorticoids are unnecessary and ACTH is not useful. CT scan would be indicated if the patient did not respond to fluid and steroids and adrenal insufficiency is ruled out. Angiography and surgical exploration are also premature given the clinical scenario suggestive of adrenal crisis. I.V. desmopressin would not be first line treatment in this patient with adrenal crisis, but could be indicated if hypotension was refractory to fluid resuscitation and steroids. Motzer RJ Rini BI, Bukowski RM, et al: Sunitinib in patients with metastatic renal cell carcinoma. JAMA 2006;295:2516. Vaughan ED Jr, Blumenfeld JD: Pathophysiology, evaluation, and medical management of adrenal disorders, in Wein AJ, Kavoussi LR, Novick AC, Partin AW, Peters CA (eds): CAMPBELL'S UROLOGY, ed 9. Philadelphia, Saunders Elsevier, 2007, vol 2, chap 53, pp 1844-1845.

2012 - 116 A 73-year-old woman with a serum creatinine of 1.9 mg/dl undergoes right partial nephrectomy for a 4.5 cm hilar renal mass. Frozen section of the resected mass reveals chromophobe RCC with tumor less than 0.1 mm from the margin of resection in the area of renal sinus. The next step is: A. no further therapy. B. cryotherapy of resection bed. C. radiofrequency ablation (RFA) of resection bed. D. wide resection of renal sinus tissue. E. completion radical nephrectomy.

A . The patient exhibits an absolute indication of nephron sparing surgery with a serum creatinine of 1.9 mg/dl which translates into a estimated GFR of 26 ml/min. This patient is in need of retaining as much functional renal parenchyma as possible. This coupled with the fact that she has exhibited an indolent chromophobe subtype of renal cancer is a strong indication to not perform further cancer eradicating procedures. In addition, even in the presence of a positive focal margin there is a very low risk of renal cancer relapse as demonstrated by multiple retrospective data analysis of this scenario. Energy-based therapies in the renal hilar bed would be associated with a high rate of renal unit loss. Sundaram V, Figenshau RS, Roytman TM: Positive margin during partial nephrectomy: Does cancer remain in the renal remnant? UROL 2011;77:1400-1403. Bensalah K, Pantuck AJ, Rioux-Leclercq N: Positive surgical margin appears to have negligible impact on survival of renal cell carcinomas treated by nephron-sparing surgery. EUR UROL 2010;57:466-471. Yossepowitch O, Thompson RH, Leibovich BC: Positive surgical margins at partial nephrectomy: Predictors and oncological outcomes. J UROL 2008;179:2158-2163.

2012 - 110 A 60-year-old man with erectile dysfunction has an impalpable right testicle. Ultrasound reveals a 2 x 2 cm homogeneous ovoid mass at the right internal ring. The next step is: A. observation. B. serial ultrasounds. C. percutaneous biopsy. D. orchidopexy. E. orchiectomy.

A . The peak age of onset of testicular cancer is between 25 and 35 and cryptorchidism is the only know risk factor for testicular cancer. However the risk of developing a testis tumor in an undescended testis drops dramatically after age 30. It has been calculated that the risk of dying from testis cancer after age 32 is outweighed by the risk of surgical mortality, though others have suggested a cutoff of 50. At age 60, this patient should be reassured and does not require serial ultrasounds. Rozanski TA, Bloom DA: The undescended testis. Theory and management. UROL CLIN N AM 1995;22:107-118. Richie JP, Steele GS: Neoplasms of the testis, in Wein AJ, Kavoussi LR, Novick AC, Partin AW, Peters CA (eds): CAMPBELL'S UROLOGY, ed 9. Philadelphia, Saunders Elsevier, 2007, vol 1, chap 29, p 893.

2013 - 24 The neurovascular bundles on the prostate travel between the following two layers of fascia: A. levator and prostatic. B. Denonvilliers' and levator. C. Denonvilliers' and prostatic. D. lateral pelvic and prostatic. E. lateral pelvic and levator.

A . The prostate is covered with three distinct and separate fascial layers: Denonvilliers' fascia, the prostatic fascia, and the levator fascia. Denonvilliers' fascia is a filmy, delicate layer of connective tissue located between the anterior walls of the rectum and prostate. The neurovascular bundle on the prostate contain the cavernosal nerves and are located between the layers of the levator fascia and prostatic fascia. Schaeffer EM, Partin AW, Walsh PC: Radical retropubic and perineal prostatectomy, Wein AJ, Kavoussi LR, Novick AC, Partin AW, Peters CA (eds): CAMPBELL-WALSH UROLOGY, ed 10. Philadelphia, Elsevier Saunders, 2012, vol 3, chap 102, p 2801.

2013 - 9 A 61-year-old man with a serum creatinine of 1.7 mg/dl has a 5 cm upper pole left renal mass. He undergoes left partial nephrectomy. After complete gross resection of the mass frozen section reveals lymphoma with diffuse infiltration of normal renal parenchyma by lymphoma at the margins. His cold ischemic time was 18 minutes. The next step is: A. no further operative intervention. B. re-excision of tumor bed. C. cryotherapy of margin. D. biopsy of contralateral kidney. E. radical nephrectomy.

A . The unexpected finding of renal lymphoma at the time of renal cortical tumor surgery is rare. Ninety percent of these cases are not primary renal lymphoma but rather systemic lymphoma with renal manifestation. Non-Hodgkin's lymphomas are the most common subtype. Multifocal masses, bilaterality and regional lymphadenopathy are all more common in renal lymphoma than in renal cortical tumors. In this patient the presence of diffuse renal infiltration by lymphoma will make post-operative systemic therapy necessary. In the setting of compromised renal function every attempt should be made to spare the remaining nephron mass in preparation for systemic chemotherapy. Therefore further surgical intervention is not warranted and completion of the operation and subsequent postoperative discussion regarding systemic therapy is the most logical next step. Campbell SC, Lane BR: Malignant renal tumors, Wein AJ, Kavoussi LR, Novick AC, Partin AW, Peters CA (eds): CAMPBELL-WALSH UROLOGY, ed 10. Philadelphia, Elsevier Saunders, 2012, vol 2, chap 49, pp 1471-1472.

2011 - 57 A six-year-old girl has a palpable abdominal mass. CT scan demonstrates a 5 cm mass in the lower pole of the left kidney and a 2 cm lesion in the upper pole of the right kidney. Biopsy of the left renal mass shows Wilms' tumor. The next step is: A. chemotherapy. B. chemotherapy and XRT. C. bilateral partial nephrectomies. D. left radical nephrectomy and right partial nephrectomy. E. left radical nephrectomy and chemotherapy.

A . There is current controversy as to whether patients with Wilms' tumor should undergo primary surgical exploration or initial pre-operative chemotherapy. However, there is general consensus that there are several instances when surgical therapy should not be undertaken, other than biopsy. This includes vena caval extension of the tumor above the hepatic veins, inoperable tumor, and bilateral disease. Bilateral disease is seen in about 5% of cases. In these cases, therapy is directed towards preservation of renal tissue since there is high incidence of renal failure, especially when a primary nephrectomy is undertaken. Thus, primary chemotherapy is indicated in patients with bilateral disease with a subsequent surgical exploration following completion of the first course of chemotherapy. Ritchey ML, Ferrer FA: Wilms' tumor, Docimo SG, Canning DA, Khoury AE (eds): CLINICAL PEDIATRIC UROLOGY, ed 5. London, Informa Healthcare, 2007, p 507.

2013 - 95 A 43-year-old paraplegic man with a neurogenic bladder has gross hematuria. A cystogram shows bilateral grade 2 VUR and a 1 cm papillary filling defect in the mid-right ureter. Cystoscopy shows patulous ureteral orifices, but no other abnormalities. Complete ureteroscopic resection of the tumor reveals a low grade non-invasive urothelial carcinoma. The serum creatinine is 1.0 mg/dl. The best management is: A. surveillance with ureteroscopy. B. nephroureterectomy. C. partial ureterectomy. D. ureterectomy and ileal ureter. E. intravesical BCG.

A . This patient is an ideal candidate for endourologic management of an upper tract tumor. The patulous ureteral orifices will allow easy passage of the ureteroscope for surveillance. The surgical options listed are unnecessary for a completely resected tumor. Intravesical BCG is not indicated for a first-time low grade tumor. Martinez-Pineiro JA, Matres MJG, Martinez-Pineiro L: Endourologic treatment of upper tract urothelial carcinomas: analysis of a series of 59 tumors. J UROL 1996;156:377-385. Sagalowsky AI, Jarrett TW, Flanigan RC: Urothelial tumors of the upper urinary tract and ureter, Wein AJ, Kavoussi LR, Novick AC, Partin AW, Peters CA (eds): CAMPBELL-WALSH UROLOGY, ed 10. Philadelphia, Elsevier Saunders, 2012, vol 2, chap 53, p 1516.

2014 - 113 A repeat prostate biopsy after an initial finding of atypical small acinar proliferation (ASAP), as compared to high-grade prostatic intraepithelial neoplasia (HGPIN), has: A. a significantly higher risk of prostate cancer. B. a significantly higher risk of high-grade prostate cancer only. C. an equivalent risk of prostate cancer. D. a significantly lower risk of prostate cancer. E. a significantly lower risk of high-grade prostate cancer only.

A . With regards to prostatic intraepithelial neoplasia (PIN): 1) Low-grade PIN should not be documented in pathology reports due to poor interobserver reproducibility and a relatively low risk of cancer following re-biopsy, 2) The expected incidence of HGPIN on needle biopsy is between 5-8%, 3) Although the diagnosis of HGPIN is subjective, interobserver reproducibility for its diagnosis is fairly high among urological pathologists, 4) The median risk recorded in the literature for finding of cancer sometime in the patient's life following the diagnosis of HGPIN on needle biopsy is approximately 24%, 5) The majority of publications that compared the risk of cancer following a needle biopsy diagnosis of HGPIN to the risk of cancer following a benign diagnosis on needle biopsy show no differences between the two groups, 6) Clinical and pathological parameters do not help stratify which men with HGPIN are at increased risk for a future diagnosis of prostate cancer, 7) A major factor contributing to the decreased incidence of cancer following a diagnosis of HGPIN on needle biopsy in the contemporary era is related to increased needle biopsy core sampling, which detects many associated cancers on initial biopsy, such that re-biopsy following HGPIN, even with good sampling, does not detect many additional cancers, 8) It is recommended that men do not need routine repeat needle biopsy within the first year following the diagnosis of HGPIN. Further studies are needed to confirm whether routine repeat biopsies should be performed several years following a HGPIN diagnosis. In contrast, atypical small acinar proliferation (ASAP) is a small foci of glands that exhibit some features of adenocarcinoma, it is believed this diagnosis is frequently due to insufficient biopsy material to make the diagnosis of prostate cancer and therefore repeat biopsy is recommended. Regarding ASAP: 1) An average of 5% of needle biopsy pathology reports are diagnosed as having ASAP, 2) Cases diagnosed as ASAP have the highest likelihood of being changed to prostate cancer upon expert review and urologists should consider sending such cases for consultation in an attempt to resolve the diagnosis as definitively benign or malignant before subjecting the patient to repeat biopsy, 3) Ancillary pathologic techniques using basal cell markers and AMACR (alpha-methyl-acyl-coenzyme A racemase) can decrease the number of ASAP diagnoses. 4) The average risk of cancer on a subsequent biopsy following a diagnosis of ASAP within a six month time period is approximately 40%, 5) Clinical and pathological parameters do not help predict which men with an atypical diagnosis have cancer on repeat biopsy, 6) Repeat biopsy should include increased sampling of the initial atypical site, and adjacent ipsilateral and contralateral sites with routine sampling of all sextant sites. Therefore, it is critical for urologists to submit needle biopsy specimens in a manner in which the sextant location of each core can be determined, 7) All men with an ASAP diagnosis need re-biopsy within three to six months. It is critical for urologists to distinguish between a diagnosis of HGPIN and that of ASAP on needle biopsy. These two entities indicate different risks of carcinoma on re-biopsy and different recommendations for followup. Epstein JI, Herawi M: Prostate needle biopsies containing prostatic intraepithelial neoplasia or atypical foci suspicious for carcinoma: Implications for patient care. J UROL 2006;175:820-834.

2012 - 104 A 67-year-old man with a clinical stage T2bN0M0 Gleason 6 prostate cancer with a PSA of 7.8 ng/ml is treated with 70 Gy external beam XRT. His PSA nadirs to 0.8 ng/ml six months after therapy. Six months later, he is asymptomatic, has a normal DRE, and a PSA of 6.5 ng/ml. The most likely explanation for the elevated PSA level is: A. prostatic infarct. B. persistent prostate cancer. C. PSA bounce effect. D. radiation-induced prostatitis. E. insufficient period of observation after therapy.

B . A marked increase in serum PSA after a nadir within six months of external beam XRT is a sign of persistent local or occult metastatic prostate cancer and has a poor prognosis. Radiation induced cellular injury or prostatitis may cause a minor rise in PSA which usually returns to normal within a few weeks. A bounce can be defined as a rise greater than 0.2 ng/ml followed by a durable decline and is especially common after brachytherapy, where it is reported to occur in 24% to 35% of men. These can start any time from 9 to 30 months after brachytherapy with the majority of patients having a cumulative PSA rise of not more than 2 to 3 ng/ml. Prostatic infarct is rare following radiation therapy for prostate cancer, and would likely be associated with a significantly elevated PSA. D'Amico AV, Crook J, Beard CJ, et al: Radiation therapy for prostate cancer, in Walsh PC, Retik AB, Vaughan ED Jr, Wein AJ (eds): CAMPBELL'S UROLOGY, ed 8. Philadelphia, WB Saunders Co, 1998, chap 92, p 3147. D'Amico AV, Crook J, Beard CJ, et al: Radiation therapy for prostate cancer, in Wein AJ, Kavoussi LR, Novick AC, Partin AW, Peters CA (eds): CAMPBELL'S UROLOGY, ed 9. Philadelphia, Saunders Elsevier, 2007, vol 3, chap 100, pp 3009-3011, 3013.

2013 - 104 A 23-year-old man develops chylous ascites after RPLND. He is initially managed with a medium chain triglyceride diet, and for the last two weeks, with TPN. He still requires periodic paracentesis four weeks postoperatively. The next step is: A. continue current management. B. add somatostatin. C. laparoscopic ligation of the source. D. open surgical ligation of the source. E. peritoneovenous shunt.

B . After making the patient NPO and starting TPN, the next step is the administration of subcutaneous or I.V. somatostatin. Recent reports have shown a beneficial effect leading to drying up of lymphatic fistulas in this setting. Somatostatin works by decreasing the absorption of fats, inhibiting gastric, intestinal, and pancreatic secretions, and inhibiting motor activity of the intestines. The net effect is reduced flow within the major lymphatic channels, and reduced leakage from the fistula site. Somatostatin is thus recommended prior to proceeding with more invasive measures. Lievovitch I, Mor Y, Golomb J, Ramon J: The diagnosis and management of postoperative chylous ascites. J UROL 2002;167:449-457.

2012 - 86 A 26-year-old man undergoes a left radical orchiectomy for seminoma. CT scan reveals two 3 cm interaortocaval nodes. Two weeks after orchiectomy, the AFP is unchanged at 25 IU/ml and beta-hCG is 8 mIU/ml. The next step is: A. infradiaphragmatic XRT. B. cisplatin-based chemotherapy. C. infradiaphragmatic plus supradiaphragmatic XRT. D. RPLND. E. RPLND plus cisplatin-based chemotherapy.

B . Although the histological diagnosis is seminoma, the elevated alpha-fetoprotein is indicative of a non-seminomatous testis tumor, and the patient should be managed as such. The orchiectomy specimen should be re-evaluated in detail to look for an additional germ cell elements. Given the findings on the CT scan, this patient has a clinical Stage IIB tumor. Based on the data available in the literature, it appears these patients are best served with initial chemotherapy rather than RPLND following the inguinal orchiectomy. The most widely utilized regimen today is three cycles of bleomycin, etoposide (VP-16), and cisplatin. If residual nodal tissue is evident after the multidrug chemotherapy, surgical excision is recommended. With the evolution of effective multi-drug regimens, radiation therapy is no longer utilized in the management of Stage IIB non-seminomatous testis tumors. Richie JP, Steele GS: Neoplasms of the testis, in Wein AJ, Kavoussi LR, Novick AC, Partin AW, Peters CA (eds): CAMPBELL'S UROLOGY, ed 9. Philadelphia, Saunders Elsevier, 2007, vol 1, chap 29, pp 896-897

2012 - 75 A two-year-old boy with hemihypertrophy should undergo: A. twice yearly physical exam of abdomen. B. renal ultrasound every three months. C. twice yearly urinalysis and urinary metanephrine level. D. annual abdominal CT scan. E. abdominal MRI scan every six months.

B . Children with hemihypertrophy are at risk for the development of Wilms' tumor due to alterations in the WTR1 and WTR2 genes. The best screening method to allow early detection of a renal mass, without untoward exposure to radiation or excessive cost, is ultrasound every three months through early childhood. Twice-yearly urinalysis and urinary metanephrine are not indicated in this patient, because they are not predisposed to pheochromocytomas. Ritchey ML, Shamberger RC: Pediatric urologic oncology, in Wein AJ, Kavoussi LR, Novick AC, Partin AW, Peters CA (eds): CAMPBELL'S UROLOGY, ed 9. Philadelphia, Saunders Elsevier, 2007, vol 4, chap 130, p 3904.

2013 - 143 A 55-year-old asymptomatic man with a serum PSA of 5.0 ng/ml has a normal DRE. TRUS is normal and prostate biopsy reveals Gleason 3+3 adenocarcinoma in two biopsy cores from the left base and two biopsy cores from the right apex. The remaining biopsies show benign prostate tissue. According to the 2010 AJCC TNM classification, the clinical T stage is: A. cT1b. B. cT1c. C. cT2a. D. cT2b. E. cT2c.

B . Clinical staging is determined by DRE and TRUS, and is not influenced by biopsy location information. This is a common clinical mistake by practicing urologists. The correct clinical stage is cT1c since it is a PSA diagnosed lesion with a normal physical examination and a normal ultrasound. If he had a small nodule on physical examination or consistently present on ultrasound (or other imaging) he would be cT2a. If the nodule was bilateral his clinical stage would be cT2c. If he has a radical prostatectomy, his most likely pathologic stage will be pT2c since his disease is bilateral on biopsy. Loeb S, Carter HB: Early detection, diagnosis, and staging of prostate cancer, Wein AJ, Kavoussi LR, Novick AC, Partin AW, Peters CA (eds): CAMPBELL-WALSH UROLOGY, ed 10. Philadelphia, Elsevier Saunders, 2012, vol 3, chap 99, pp 2767-2768.

2014 - 132 The site of origin associated with the worst prognosis in pediatric rhabdomyosarcoma is: A. uterus. B. prostate. C. vagina. D. spermatic cord. E. bladder.

B . Combination chemotherapy with or without extirpative surgery has improved the prognosis for patients with rhabdomyosarcoma. The prostate has the worst prognosis as pertains to organ of origin, and radical surgery may be required if there is lack of response to non-operative treatment. The best prognosis in rhabdomyosarcoma has been associated with vaginal origin and embryonal histology. Ritchey ML, Shamberger RC: Pediatric urologic oncology, Wein AJ, Kavoussi LR, Novick AC, Partin AW, Peters CA (eds): CAMPBELL-WALSH UROLOGY, ed 10. Philadelphia, Elsevier Saunders, 2012, vol 4, chap 137, p 3704.

2012 - 12 During radical cystectomy, the cephalad (proximal) limit of an extended pelvic lymph node dissection is the: A. aortic bifurcation. B. inferior mesenteric artery. C. bifurcation of the common iliac artery. D. the genitofemoral nerve. E. the circumflex iliac vein.

B . Extended pelvic lymph node dissection has been associated with an improved disease specific survival in patients with muscle invasive bladder cancer. Increasing the number of lymph nodes removed at lymph node dissection is associated with improved survival in the setting of both lymph node negative and positive disease. The cephalad limit of the extended pelvic lymph node dissection for bladder cancer is the inferior mesenteric artery. The bifurcation of the common iliac artery is the cephalad limit of dissection of the standard pelvic lymphadenectomy. The circumflex iliac vein and genitofemoral nerve are the caudal and lateral limits of dissection. Bochner BH, Carver BS, Coleman JA, Eastham JA: Role of lymphadenectomy in genitourinary cancer. AUA UPDATE SERIES 2009; vol 28, lesson 27, pp 251-252.

2011 - 3 A 25-year-old man has a thickened, indurated fat mass excised from his spermatic cord at the time of inguinal hernia repair. Final pathology reveals low grade liposarcoma with negative margins. The next step is: A. observation. B. inguinal orchiectomy. C. inguinal orchiectomy and RPLND. D. inguinal orchiectomy and hemiscrotectomy. E. inguinal/abdominal radiation.

B . Liposarcoma of the paratesticular structures is most often associated with the spermatic cord. it is a rare tumor that is usually well-differentiated. As with all sarcomas of the paratesticular region, inguinal orchiectomy with high ligation of the spermatic cord with inguinal orchiectomy is generally advised to minimize the chance of local recurrence. Because of the low likelihood of hematogenous or lymphatic spread in a low grade sarcoma, additional radiation or chemotherapy would not likely be necessary. This tumor has a low likelihood of complete response to primary radiotherapy and therefore it is not a reasonable option. Hemiscrotectomy is unnecessary with no violation of the scrotum. Richie JP, Steele GS: Neoplasms of the testis, in Wein AJ, Kavoussi LR, Novick AC, Partin AW, Peters CA (eds): CAMPBELL-WALSH UROLOGY, ed 9. Philadelphia, Elsevier, 2007, chap 29, p 934.

2014 - 129 In a three-year-old boy, a partial nephrectomy is the most appropriate treatment for: A. congenital mesoblastic nephroma. B. stage V Wilms' tumor. C. stage I rhabdoid tumor. D. stage III lymphoma. E. stage III neuroblastoma.

B . Mesoblastic nephromas are infiltrative tumors not amenable to partial nephrectomy. Rhabdoid tumors are highly malignant and similarly are best treated by radical nephrectomy. Lymphomas are optimally treated with systemic chemotherapy and neuroblastomas rarely involve the kidney. If the neuroblastoma involves the renal hilum, radical nephrectomy is often necessary. In the treatment of Stage V (bilateral) Wilms' tumor, renal preservation is important and hence partial nephrectomy is preferred when appropriate (mass amenable to partial nephrectomy) following chemotherapy. Ritchey ML, Shamberger RC: Pediatric urologic oncology, Wein AJ, Kavoussi LR, Novick AC, Partin AW, Peters CA (eds): CAMPBELL-WALSH UROLOGY, ed 10. Philadelphia, Elsevier Saunders, 2012, vol 4, chap 137, p 3721.

2013 - 94 A ten-year-old girl with a history of bilateral high grade VUR was treated with bilateral cross-trigonal ureteral reimplantation surgery at two years of age. She is evaluated for persistent gross hematuria following treatment of a UTI. Cystoscopy and bladder mass resection demonstrates nephrogenic adenomas. The next step is: A. reassurance. B. antibiotic suppression. C. intravesical BCG. D. laser ablation. E. partial cystectomy.

B . Nephrogenic adenoma is a rare benign metaplastic response of urothelium to tissue injury. Most nephrogenic adenomas occur after an inciting event such as surgery, trauma, infection, and immunosuppression or in response to calculi. The adenomas will develop months to years after a precipitating event and will usually occur within the bladder but may occur on any urothelial surface including transposed bladder mucosal grafts. The main presentation is hematuria and irritative voiding symptoms, but patients may also present with obstructive symptoms or be diagnosed incidentally. The endoscopic appearance is that of a papillary exophytic lesion resembling a low-grade urothelial carcinoma. On histological evaluation, nephrogenic adenoma appear as subepithelial tubular structures similar to Henle's loops. Although at one time these lesions were considered to be premalignant in nature, recent studies have been unable to establish a relationship between nephrogenic adenomas and the subsequent development of malignancy. Even though there is no evidence of malignant potential, transurethral resection is recommended together with long-term antibiotic prophylaxis for at least one year after resection. Prolonged antibiotic therapy is suggested due to the frequent finding of UTI as an associated or causative factor. It is controversial regarding whether patients with nephrogenic adenomas should undergo surveillance cystoscopy. Although there is high incidence of recurrence (30-40%) it is feared that repeated cystoscopic evaluations could further traumatize the bladder urothelium leading to an increased incidence of recurrence. Since nephrogenic adenomas are now known to be a benign condition most authorities recommend cystoscopy, only if recurrent gross hematuria and/or irritative or obstructive voiding symptoms develop. Frimberger DC, Kropp BP: Bladder anomalies in children, Wein AJ, Kavoussi LR, Novick AC, Partin AW, Peters CA (eds): CAMPBELL-WALSH UROLOGY, ed 10. Philadelphia, Elsevier Saunders, 2012, vol 4, chap 125, p 3387. Garcia-Penit J, Orsola A, Parada R, et al: Synchronous nephrogenic adenoma in the bladder and neourethra (bladder mucosa) in a boy. BR J UROL INT 1999;84:169-170. Peeker R, Aldenborg F, Fall M: Nephrogenic adenoma: A study with special reference to clinical presentation. BR J UROL 1997;80(4):539-542.

2012 - 146 During open inguinal hernia repair, a 19-year-old man undergoes complete excision of an incidental 2 cm spermatic cord mass. The final pathology shows well-differentiated leiomyosarcoma with negative surgical margins. A metastatic survey is negative. The next step is: A. surveillance. B. inguinal orchiectomy with high ligation of the cord. C. XRT to the inguinal region. D. RPLND. E. systemic chemotherapy.

B . Paratesticular leiomyosarcoma occurs most commonly during the first two decades of life. On gross inspection, these tumors appear circumscribed, but on microscopic examination they often extend well beyond the margin seen by the naked eye. Despite achieving a negative margin, local recurrence risk is significant. Because of this, the primary paratesticular tumor should be removed by inguinal orchiectomy with high ligation of the cord. Excision of the inguinal scar is also usually performed at time of orchiectomy to reduce local recurrence. Chemotherapy and radiation are reserved for patients with gross or microscopic residual disease after surgical resection. RPLND for stage I paratesticular leiomyosarcoma is not indicated. Richie JP, Steele GS: Neoplasms of the testis, in Wein AJ, Kavoussi LR, Novick AC, Partin AW, Peters CA (eds): CAMPBELL'S UROLOGY, ed 9. Philadelphia, Saunders Elsevier, 2007, vol 1, chap 29, p 933.

2014 - 128 A 62-year-old man undergoes TURBT for a solitary low-grade 1 cm papillary Ta bladder tumor. He receives a single perioperative dose of mitomycin C. This therapy will most likely decrease his risk of: A. postoperative side effects. B. short term recurrence (< 2 years). C. long term recurrence (> 2 years). D. progression. E. cancer-specific mortality.

B . Perioperative mitomycin C is effective in decreasing the chance of recurrence in short term (one to two years), however there is no effect on progression or long term recurrence rate, and no significant differences in complications has been reported when compared to other intravesical therapies. Hall MC, Chang SS, Dalbagni G, et al: Guideline for the management of nonmuscle invasive bladder cancer (stages Ta, T1, and Tis): 2007 update. BLADDER CANCER. American Urological Association Education and Research, Inc, 2007. http://www.auanet.org/education/guidelines/bladder-cancer.cfm

2012 - 122 Ductal carcinoma of the prostate: A. is best managed with chemotherapy. B. is associated with high grade disease and recurrence. C. confers no additional risk. D. should be graded as Gleason grade 5. E. commonly arises from the transition zone.

B . Prostatic ductal carcinoma is an adenocarcinoma that arises in prostatic ducts. It should be graded as a Gleason 4+4 since it shares a cribriform pattern and is associated with high grade disease and recurrence. It should be treated similarly to other high grade adenocarcinomas with combination hormonal therapy/ radiation therapy or radical prostatectomy or cryotherapy or primary hormonal therapy depending on patient age, fitness and preference. Like other carcinomas, ductal carcinoma arises more commonly in the peripheral than the transitional zone. Epstein JI: Pathology of prostatic neoplasia, in Wein AJ, Kavoussi LR, Novick AC, Partin AW, Peters CA (eds): CAMPBELL'S UROLOGY, ed 9. Philadelphia, Saunders Elsevier, 2007, vol 3, chap 91, p 2881.

2011 - 44 A 62-year-old man undergoes a TURBT at the dome. Final pathology reveals muscle-invasive small cell carcinoma. Metastatic work-up is negative. The next step is: A. restaging TURBT. B. neoadjuvant cisplatin-based chemotherapy. C. XRT. D. partial cystectomy. E. radical cystoprostatectomy.

B . Small cell carcinoma of the bladder is a relatively rare tumor of the bladder that may arise in combination with urothelial carcinoma. It is usually biologically aggressive with early vascular and muscular invasion. These malignancies usually respond to, but are not cured by, cisplatin-based chemotherapy regimens. Neither partial cystectomy nor intravesical chemotherapy is appropriate in this setting. Radiation or extirpative surgery alone may result in cure rates of 5 to 20%. However, neoadjuvant chemotherapy followed by surgery or radiation therapy results in cure rates of 40 to 65%. Therefore, the best treatment is cisplatin-based chemotherapy followed by an aggressive local treatment such as surgery or radiation if the patient does not progress. Messing EM: Urothelial tumors of the bladder, in Wein AJ, Kavoussi LR, Novick AC, Partin AW, Peters CA (eds): CAMPBELL'S UROLOGY, ed 9. Philadelphia, Saunders Elsevier, 2007, vol 3, chap 75, p 2444.

2012 - 30 A 75-year-old man underwent nephroureterectomy for pT3N0Mx ureteral cancer six weeks ago. He is now considered for adjuvant chemotherapy. He lives independently and maintains his own home; however, he cannot perform strenuous activity. He is out of bed most of the day. His ECOG (Eastern Cooperative Oncology Group) Performance Status is: A. 0 B. 1 C. 2 D. 3 E. 4

B . The ECOG Performance Status, also called the WHO or Zubrod score runs from 0 to 5, with 0 denoting perfect health and 5 death. 0 - Fully active, without restriction. 1 - Completely ambulatory, but restricted in physically strenuous activity. Able to carry out work of a light or sedentary nature such as light housework or office work. 2 - Ambulatory and capable of all self care but unable to carry out any work activities. Out of bed > 50% of waking hours. 3 - Capable of only limited self-care and confined to a bed or chair > 50% of waking hours. 4 - Completely disabled. Cannot carry on any self-care. Totally confined to bed or chair 5 - Death Oken MM, Creech RH, Tormey DC, et al: Toxicity And Response Criteria Of The Eastern Cooperative Oncology Group. AM J CLIN ONCOL 1982;5:649-655. Galsky MD, Hahn NM, Rosenberg J, et al: Treatment of patients with metastatic urothelial cancer unfit for Cisplatin-based chemotherapy. J CLIN ONCOL 2011;29:2432-2438.

2012 - 107 A 50-year-old woman has acute left flank pain. Her hemoglobin is 7 gm/dl, and creatinine 2.1 mg/dl. CT scan shows a 15 cm lower-pole left renal mass and a large perinephric hematoma. There are low-density areas (Hounsfield units of -30) within the mass. After transfusion, the next step is: A. observation. B. selective angiographic embolization. C. radical nephrectomy. D. partial nephrectomy. E. radio frequency ablation.

B . The negative Hounsfield units on the CT scan indicate the presence of fat in the tumor, showing that this is a renal angiomyolipoma which has bled acutely. Angiographic embolization is often successful in these tumors, can be performed quickly without the need for general anesthetic, and can usually preserve the normal portion of the kidney. If a patient has tuberous sclerosis, bilateral disease, preexisting renal insufficiency, or other medical or urologic disease that could affect renal function in the future, selective embolization should be considered as the treatment. An alternative would be lower pole partial nephrectomy, once the patient is stabilized, but not in the acute setting as often times radical nephrectomy would be required. This would put this patient at significantly increased risk of dialysis. Laparoscopic nephrectomy would be very difficult in the face of recent acute hemorrhage. Radio frequency ablation should not be used for such a large tumor. Campbell SC, Novick AD, Bukowski RM: Renal tumors, in Wein AJ, Kavoussi LR, Novick AC, Partin AW, Peters CA (eds): CAMPBELL'S UROLOGY, ed 9. Philadelphia, Saunders Elsevier, 2007, vol 2, chap 47, p 1580.

2012 - 33 A 68-year-old woman on recommended daily allowance (RDA) vitamins develops CIS of the bladder and is starting intravesical immunotherapy. In an effort to maximize tumor response rates and minimize side effects she should receive induction and subsequent maintenance therapy with BCG and: A. discontinue vitamins. B. continue RDA vitamins. C. initiate mega dose vitamin supplements. D. interferon, and RDA vitamins. E. interferon, and mega dose vitamin supplements.

B . The use of both vitamin supplements and alpha interferon has become common in conjunction with BCG therapy. Recently a randomized prospective trial of 670 patients was completed to address these issues. BCG alone with RDA vitamins was associated with an equal disease free outcome when compared to combinations of interferon or mega dose vitamin supplements. Megadose vitamins showed no increased benefits when compared with RDA vitamins. All patients were treated with induction and maintenance BCG. Patients receiving interferon experienced greater side effects, particularly fever and constitutional symptoms. There is no compelling evidence to suggest that patient should discontinue RDA vitamins. Nepple KG, Lightfoot AJ, Rosevear HM, et al: Bladder Cancer Genitourinary Oncology Study Group. Bacillus Calmette-Guerin with or without interferon alpha-2b and megadose versus recommended daily allowance vitamins during induction and maintenance intravesical treatment of nonmuscle invasive bladder cancer. J UROL 2010;184:1915-1919.

2013 - 128 A 66-year-old man has a PSA of 4.1 ng/ml and is diagnosed with a Gleason sum 6 prostate cancer involving less than 5% of a single core from a 12-core biopsy. He elects active surveillance. One year later, his PSA is 4.5 ng/ml. The next step is: A. recheck PSA in three months. B. recheck PSA in six months. C. 12-core TRUS-guided prostate biopsy. D. 24-core TRUS-guided saturation biopsy. E. MRI.

C . Active surveillance is an appropriate management strategy for selected men with localized prostate cancer. Patients who elect this approach should have favorable risk tumors, usually consisting of low-volume Gleason sum 6 or less disease. Active surveillance protocols are characterized by close monitoring of PSA kinetics and serial biopsy. Klotz et al from Toronto have suggested that PSAs be measured quarterly for the first two years after diagnosis and then twice annually after that, assuming the PSA is stable. A PSA doubling time of less than three years is considered to be an indication for intervention. They have recommended a 10-12 core biopsy at one year after original diagnosis and then every three to five years after that until age 80. If any Gleason sum 7 or higher disease is noted, this is considered to be an indication for intervention. In this case, the patient is one year removed from his initial diagnosis and a follow-up biopsy is indicated. Reassessment of PSA in three or six months is inappropriate if repeat biopsy is not performed. Saturation biopsy is not indicated in this setting. MRI can be useful in the follow-up for patients, but is not a critical element of active surveillance protocols. Klotz L: Active surveillance for prostate cancer: Patient selection and management. AUA UPDATE SERIES 2008 vol 27, lesson 33, p 326.

2014 - 126 A three-year-old girl has an abdominal mass and periorbital ecchymoses. These findings are most suggestive of: A. tuberous sclerosis. B. Wilms' tumor. C. neuroblastoma. D. Hodgkin's disease. E. leukemia.

C . After brain tumors, neuroblastoma is the most common malignant solid tumor of childhood. More than half of these originate in the abdomen and a palpable abdominal mass is the most common finding. Periorbital metastatic disease is common, causing periorbital edema, proptosis, or raccoon-like ecchymoses. Tuberous sclerosis is associated with facial lesions of adenoma sebaceum in 75% of cases. These lesions are firm, discrete, red or brown telangiectatic papules (facial angiofibromata), and are typically located on the nasolabial folds, chin, and cheeks. Renal angiomyolipomas occur in 40-80% of patients with tuberous sclerosis. Wilms' tumor typically presents as an abdominal mass or with gross hematuria. Hodgkin's disease and leukemia often present with fever, night sweats, or fatigue. Neither Wilms' tumor, Hodgkin's disease, nor leukemia are associated with facial or periorbital lesions. Pope JC IV: Renal dysgenesis and cystic disease of the kidney, Wein AJ, Kavoussi LR, Novick AC, Partin AW, Peters CA (eds): CAMPBELL-WALSH UROLOGY, ed 10. Philadelphia, Elsevier Saunders, 2012, vol 4, chap 118, p 3177.

2011 - 30 A 47-year-old uncircumcised married man is diagnosed with high grade Ta squamous cell carcinoma of the foreskin. His wife should undergo: A. observation. B. HPV vaccination. C. Pap smear. D. imiquimod therapy. E. cervix biopsy.

C . HPV infection is associated with the development of penile cancer. Wives or ex-wives of men with penile cancer have a threefold higher risk of cervical carcinoma. The male partners of women with cervical intraepithelial neoplasia have a significantly higher incidence of penile intraepithelial neoplasia. Therefore, screening with Pap smear and pelvic exam is prudent in this setting. HPV vaccination is inappropriate because it is only effective prior to exposure. More aggressive therapy such as biopsy or topical therapy is inappropriate unless a diagnosis of cervix cancer is suspected on physical exam and Pap smear. Pettaway CA, Lynch DF Jr, Davis JW: Tumors of the penis, in Wein AJ, Kavoussi LR, Novick AC, Partin AW, Peters CA (eds): CAMPBELL'S UROLOGY, ed 9. Philadelphia, Saunders Elsevier, 2007, vol 1, chap 31, p 965.

2014 - 146 A 44-year-old woman undergoes a left radical nephrectomy for a 6 cm RCC with renal vein involvement. The posterior surgical margin is positive. Pathologic tumor stage is: A. T1b. B. T2b. C. T3a. D. T3b. E. T4.

C . If this 6 cm tumor were contained within GerotaÆs fascia, the pathologic (p)T stage would be pT1b. However, there is renal vein involvement that is now classified as pT3a (previously pT3b). Tumors with isolated renal vein thrombus are known to have a relatively favorable prognosis, and as a result are now staged as T3a rather than T3b. The margin status is designated as R1, but does not affect primary pathologic tumor stage. Edge SB, Byrd D, Compton C, Fritz A: AJCC Staging Manual, ed 7. New York, Springer-Verlag, 2010, chap 43, pp 479-490.

2012 - 9 A 56-year-old man undergoes partial penectomy for pT2 squamous cell carcinoma. Examination reveals no inguinal adenopathy. The primary tumor characteristic most predictive of pathologic lymph node involvement is: A. HPV status. B. tumor thickness > 5 mm. C. lymphovascular invasion. D. corpora spongiosum involvement. E. corpora cavernosal involvement.

C . In a multi-institutional review, eight different factors including superficial growth pattern, grade, tumor thickness, involvement of corporal tissue and the urethra increased the likelihood of pathologic lymph node involvement. However, the factor most predictive of lymph node involvement was the presence of lymphatic and/or vascular invasion seen in the primary tumor. Although the presence of human papilloma virus infection in patients with penile cancer is common, there is no correlation to lymph node metastases risk. Novara G, Galfano A, De Marco V, et al: Prognostic factors in squamous cell carcinoma of the penis. NATURE CLIN PRACTICE UROL 2007;4:140-144.

2014 - 24 A 60-year-old smoker has a highly suspicious voided urinary cytology. CT urogram is normal. Cystoscopy, bladder biopsy, and bilateral retrograde pyelograms are normal. Selective left upper tract cytologies are highly suspicious for malignancy. The next step is: A. repeat cystoscopy, biopsy, retrograde pyelography, and selective cytologies in three months. B. repeat left ureteral washings for fluorescent in-situ hybridization (FISH). C. left ureteropyeloscopy. D. left ureteral stent and intravesical BCG. E. left percutaneous nephrostomy and antegrade BCG therapy.

C . In cases of unilateral upper tract cytologic abnormalities (with normal cystoscopy, pyelography, and bladder biopsies), ureteropyeloscopy is indicated as the next step. Ureteropyeloscopy allows for direct visualization of small lesions and is superior to retrograde pyelography in the detection of small tumors. Biopsy at the time of ureteropyeloscopy should be attempted, if feasible. A persistently abnormal cytology without any visualized lesions may signify CIS. In the past, nephroureterectomy was performed for a unilateral cytologic abnormality of the upper tract to eliminate presumed CIS, but this is no longer considered an appropriate initial approach. Observation is also not appropriate without further evaluation given the repeated abnormal cytologies. Current approaches for presumed upper tract CIS include topical immunotherapy or chemotherapy, delivered retrograde intravesically with ureteral stents in place to assist with reflux or antegrade via a nephrostomy tube under careful pressure control. Novel urinary markers (e.g., FISH) have been reported for upper tract tumor surveillance and may aid in the detection of such tumors. To date, none of these markers have a high enough sensitivity or specificity to make decisions for or against therapeutic intervention. In this case, repeat procedure under anesthesia to obtain a selective urinary sample for FISH is not warranted. Sagalowsky AI, Jarrett TW, Flanigan RC: Urothelial tumors of the upper urinary tract and ureter, Wein AJ, Kavoussi LR, Novick AC, Partin AW, Peters CA (eds): CAMPBELL-WALSH UROLOGY, ed 10. Philadelphia, Elsevier Saunders, 2012, vol 2, chap 53, pp 1549-1550. Wood DP: Urothelial tumors of the bladder, Wein AJ, Kavoussi LR, Novick AC, Partin AW, Peters CA (eds): CAMPBELL-WALSH UROLOGY, ed 10. Philadelphia, Elsevier Saunders, 2012, vol 3, chap 80, pp 2328-2329.

2014 - 140 The most common site of origin of extragonadal germ cell tumors is the: A. lung. B. pelvis. C. mediastinum. D. pineal gland. E. retroperitoneum.

C . In decreasing order of frequency, the most common sites of origin of extragonadal germ cell tumors are the mediastinum, retroperitoneum, sacrococcygeal region, and pineal gland, although many unusual sites have also been reported. Stephenson AJ, Gilligan TD: Neoplasms of the testis, Wein AJ, Kavoussi LR, Novick AC, Partin AW, Peters CA (eds): CAMPBELL-WALSH UROLOGY, ed 10. Philadelphia, Elsevier Saunders, 2012, vol 1, chap 31, p 845.

2013 - 3 A 66-year-old woman has a polypoid mass at her urethral meatus. Office biopsy demonstrates a non-invasive high-grade squamous cell carcinoma. Radiologic staging reveals no evidence of regional or distant disease. The next step is: A. topical 5-FU cream. B. laser fulguration. C. circumferential excision including excision of anterior vaginal wall. D. XRT with ilioinguinal lymphadenectomy. E. anterior pelvic exenteration with pelvic lymphadenectomy.

C . In female urethral cancers, treatment recommendations are primarily dependent on tumor location and clinical stage. Local excision may be sufficient for the relatively uncommon small, superficial, distal urethral tumors, and can result in excellent functional results. For more proximal and advanced urethral tumors, a more aggressive approach is warranted. Small, exophytic, superficial tumors arising from the urethral meatus or anterior urethra (as in this case) may be surgically treated with circumferential excision of the distal urethra including a portion of the anterior vaginal wall. The distal third of the urethra may be excised while still maintaining urinary continence. Tumors in the distal urethra tend to be low stage, and cure rates of 70% to 90% have been achieved with local excision alone. 5-FU cream has been utilized in the treatment of carcinoma-in situ of the penis, but does not have a defined role in female urethral cancers. Although XRT has been effectively used for female urethral cancers, the addition of prophylactic lymphadenectomy in this choice makes it incorrect. Recommendations for performing groin dissection have been made only for patients who present with positive inguinal or pelvic lymphadenopathy without distant metastasis, or patients who develop adenopathy during surveillance. Anterior pelvic exenteration is employed for patients with proximal urethral cancers often as part of a multimodal approach including chemotherapy and XRT. Sharp DS, Angermeier KW: Surgery of penile and urethral carcinoma, Wein AJ, Kavoussi LR, Novick AC, Partin AW, Peters CA (eds): CAMPBELL-WALSH UROLOGY, ed 10. Philadelphia, Elsevier Saunders, 2012, vol 1, chap 35, p 934.

2014 - 21 A 50-year-old smoker with gross hematuria has a 1 cm left mid-ureteral filling defect on CT urography. The lesion is biopsied and laser ablated ureteroscopically. Histology reveals an inverted papilloma. The next step is: A. no further treatment or follow-up. B. long-term antibiotics. C. surveillance of the bladder and upper tracts. D. segmental ureterectomy. E. left nephroureterectomy.

C . Inverted papillomas are typically of two types (Type 1 and Type 2). Type 2 may exhibit a malignant behavior whereas Type 1 is benign. Unfortunately, they are histologically indistinguishable. Because of this, conservative treatment followed by surveillance for two years is recommended. Therefore no further treatment or follow-up is incorrect. Since it is possibly a benign lesion and presumably completely ablated by the laser, no additional treatment is warranted including antibiotic therapy. Sagalowsky AI, Jarrett TW, Flanigan RC: Urothelial tumors of the upper urinary tract and ureter, Wein AJ, Kavoussi LR, Novick AC, Partin AW, Peters CA (eds): CAMPBELL-WALSH UROLOGY, ed 10. Philadelphia, Elsevier Saunders, 2012, vol 2, chap 53, p 1521.

2012 - 143 The factor most predictive of finding fibrosis only during post-chemotherapy RPLND for germ cell tumor is: A. use of bleomycin chemotherapy regimen. B. size of pre-chemotherapy retroperitoneal mass. C. normal post-chemotherapy CT scan. D. teratoma in primary tumor. E. normal preoperative serum tumors markers.

C . Normal serum tumor markers following chemotherapy are associated with a 10-20% chance of viable germ cell tumor and 30% probability of teratoma. Normal CT scan with no lymph node larger than 8 mm is associated with a greater than 90% chance of fibrosis at the time of RPLND. The presence or absence of bleomycin in the chemotherapy regimen is not associated with probability of response and the prechemotherapy mass size or stage is not a powerful predictor of post chemotherapy histology. Normal pre-chemotherapy tumor markers and the presence of teratoma are not predictive of fibrosis at RPLND. Sheinfeld J, Bartsch G, Bosl GJ: Surgery of testicular tumors, in Wein AJ, Kavoussi LR, Novick AC, Partin AW, Peters CA (eds): CAMPBELL'S UROLOGY, ed 9. Philadelphia, Saunders Elsevier, 2007, vol 1, chap 30, p 936.

2013 - 39 In a patient with muscle invasive urothelial carcinoma, the pathologic characteristic that predicts a poor response to neoadjuvant chemotherapy is: A. squamous differentiation. B. lymphovascular invasion. C. micropapillary variant. D. p53 mutation. E. small cell component.

C . Patients with histologic variants of urothelial carcinoma including squamous differentiation and small cell component actually appear to respond better to neoadjuvant chemotherapy. Similarly patients with a p53 mutation and lymphovascular invasion are considered higher risk patients with urothelial cancer and are recommended to have neoadjuvant chemotherapy. Micropapillary variant of urothelial carcinoma is the one variant that does not appear to respond to chemotherapy and requires immediate cystectomy. Scosyrev E, Ely BW, Messing EM, et al: Do mixed histological features affect survival benefit from neoadjuvant platinum-based combination chemotherapy in patients with locally advanced bladder cancer? A secondary analysis of Southwest Oncology Group-Directed Intergroup Study (S8710). BJU INT 2011;108:693-699. Kamat AM, Dinney CP, Gee JR, et al: Micropapillary bladder cancer: A review of the University of Texas M. D. Anderson Cancer Center experience with 100 consecutive patients. CANCER 2007;110:62-67.

2012 - 21 A 55-year-old man had a negative TRUS guided 10-core prostate biopsy two years ago for a PSA of 5 ng/ml. Now his PSA is 7 ng/ml. The next step is: A. endorectal MRI scan prior to biopsy. B. 12-core biopsies. C. 12-core biopsies including anterior apical horn biopsies. D. 12-core biopsies including transition zone biopsies. E. saturation biopsies with patient under anesthesia.

C . Repeat TRUS directed biopsies for a man with a prior negative biopsy should be at least 12 cores and should include anterior apical horn biopsies. The likelihood of a positive biopsy using this technique is between 35-50% and is similar to the yield of saturation biopsy techniques. Transition zone biopsies are also helpful but less critical than anterior apical biopsies. MRI scan can be helpful to direct repeat prostate biopsies but is expensive and is usually not the next step. Anterior apical biopsies can be obtained by transrectal or perineal approach. Carter HB, Allaf ME, Partin AW: Diagnosis and staging of prostate cancer, in Wein AJ, Kavoussi LR, Novick AC, Partin AW, Peters CA (eds): CAMPBELL'S UROLOGY, ed 9. Philadelphia, Saunders Elsevier, 2007, vol 3, chap 94, p 2912. Meng MV, Franks JH, Presti JC Jr, Shinohara K: The utility of apical anterior horn biopsies in prostate cancer detection. UROL ONC 2003;21:361-365. Presti JC Jr: Repeat prostate biopsy--when, where, and how. UROL ONC 2009;27:312-314.

2014 - 3 A patient has a transscrotal orchiectomy for a 3 cm testicular mass. At the time of exploration, there is violation of the tunica vaginalis. Pathology is pure seminoma. Chest and abdominal CT scan and serum markers are negative. The next step is: A. surveillance. B. retroperitoneal XRT. C. retroperitoneal XRT including the groin and hemiscrotum. D. excision of scrotal scar and retroperitoneal XRT. E. three cycles of BEP.

C . Suboptimal approaches to testicular neoplasms, including scrotal orchiectomy, transscrotal biopsy, or fine-needle aspiration are reported from 4-17% of the time. A recent meta-analysis of 206 cases of scrotal violation reported a local recurrence rate of 2.9% compared with 0.4% of patients treated by inguinal orchiectomy, but no difference in systemic relapse or survival rates. There did not appear to be any advantage to adjuvant therapy. Others have reported an increased local recurrence rate in patients with scrotal contamination and an 11% presence of tumor in hemiscrotectomy specimens of patients with scrotal violation. Therefore, for patients with scrotal violation with low-stage seminoma, the radiation portals should be extended to include the ipsilateral groin and scrotum. This may result in an increased risk of azoospermia. Observation is not appropriate in the setting of scrotal violation. Platinum, etoposide, and bleomycin (BEP) is appropriate for treatment of non-seminomatous germ cell tumors and advanced seminoma, not low-stage seminoma. Sheinfeld J, Bosl GJ: Surgery of testicular tumors, Wein AJ, Kavoussi LR, Novick AC, Partin AW, Peters CA (eds): CAMPBELL-WALSH UROLOGY, ed 10. Philadelphia, Elsevier Saunders, 2012, vol 1, chap 32, p 873.

2012 - 15 A 65-year-old man develops lung and liver metastases four months after undergoing a left radical nephrectomy for clear cell carcinoma. Hemoglobin is 8.1 g/dl, Creatinine is 1.3 mg/dl, and his calcium is 13 mg/dl. The therapy most likely to improve survival is: A. interferon-alpha. B. interleukin 2. C. temsirolimus. D. sunitinib. E. bevacizumab.

C . Temsirolimus is an inhibitor of the mammalian target of rapamycin (mTOR) kinase û this is a component of intracellular signaling pathways involved in growth/proliferation of cells. This medication suppresses angiogenesis and is given as a weekly IV infusion. Patients, such as this patient with three or more of the following poor risk features (serum LDH > 1.5 times upper limit of normal, Hgb below lower limit of normal, serum calcium level of more than 10 mg/dl, time from initial diagnosis of renal cell carcinoma to randomization of less than one year, Karnofsky performance of 60 or 70, or metastases in multiple organs), were found to benefit from temsirolimus. Patients who received temsirolimus were 27% more likely to survive than those who received interferon-alpha. The other listed agents have not demonstrated a survival advantage in this group of higher risk patients. Hudes G, Carducci M, Tomczak P, et al: Temsirolimus, interferon alfa, or both for advanced renal cell carcinoma. NEJM 2007;356:2271-2281.

2014 - 122 Instillation of perioperative intravesical chemotherapy after a complete TURBT for urothelial cell carcinoma of the bladder is effective only if given: A. immediately following the resection. B. within six hours of the resection. C. within 24 hours of the resection. D. within seven days of the resection. E. within two weeks of the resection.

C . The administration of a single perioperative dose of intravesical chemotherapy after complete TURBT for Ta-T1 TCC of the bladder significantly reduces the rate of tumor recurrence. This has also been summarized in a large meta-analysis in which patients receiving perioperative chemotherapy had a recurrence rate of 36.7%, compared to 48.4% with TUR alone. In all studies where benefit was noted, the instillation was given within 24 hours, generally either immediately or within six hours following the TURBT. Several randomized trials have demonstrated significant reductions in recurrence when the intravesical chemotherapy was administered within 24 hours as compared to starting it seven days later. Thus, there is evidence that the instillation should be given on the same day as the TURBT and not later. Oosterlinck W, Sylvester RJ: Perioperative instillation of chemotherapeutic drugs, Lerner SP, Schoenberg MP, Sterberg CN (eds): TEXTBOOK OF BLADDER CANCER. Abingdon, Oxon, Taylor and Francis, 2006, chap 29, pp 330-331.

2012 - 18 When compared to age-matched controls, men treated with etoposide and platinum-based chemotherapy for NSGCT are at increased long term risk of: A. systemic infection. B. pulmonary fibrosis. C. cardiovascular disease. D. ototoxicity. E. autoimmune disease.

C . The long term toxicity of bleomycin containing chemotherapy regimens includes pulmonary fibrosis however etoposide and platinum does not appear to be associated with this toxicity. There is no chronic increase in risk of systemic infection despite a short term risk of neutropenic sepsis during therapy. Several large scale epidemiologic studies have recently concluded that men treated with either radiation therapy or systemic platinum containing chemotherapy are at significantly increased risk of developing both fatal cardiovascular events as well as secondary malignancy after extended follow-up. Fossa SD, Gilbert E, Dores GM, et al: Noncancer causes of death in survivors of testicular cancer. J NAT CA INST 2007:99:533-544. Travis LB, Fossa SD, Schonfeld SJ, et al: Second cancers among 40,576 testicular cancer patients: Focus on long-term survivors. J NAT CA INST 2005;97:1354-1365.

2014 - 18 An adverse prognostic feature not included in the Motzer Criteria for patients with metastatic RCC is: A. Karnofsky performance status (KPS) < 80%. B. elevated LDH. C. thrombocytopenia. D. prior nephrectomy. E. hypercalcemia.

C . The natural history of RCC is highly unpredictable. For instance, approximately 5% of patients with what are usually small indolent tumors (< 4 cm in size), will have metastatic disease at presentation and subsequently an elevated risk of disease-specific mortality. In contrast, up to 40% of patients with lymph node metastases diagnosed at the time of nephrectomy are alive five years after surgery. The Motzer criteria is a validated risk measurement tool that the physician may employ to aid in the discussion regarding the patient's prognosis and is a useful guide in the formulation of treatment options. RCC risk groups are determined by the number of existing adverse features. The adverse features included in the Motzer criteria are Karnosky performance status < 80%, elevated LDH, anemia, hypercalcemia, and prior partial or total nephrectomy. Thrombocytopenia is not part of the criteria. If no risk factors are present, the patient is considered at low risk for recurrence. The presence of one to two features indicates intermediate risk and the presence of three to five adverse features poor/high risk for tumor recurrence. Srinivasan R, Linehan WM: Treatment of advanced renal cell carcinoma, Wein AJ, Kavoussi LR, Novick AC, Partin AW, Peters CA (eds): CAMPBELL-WALSH UROLOGY, ed 10. Philadelphia, Elsevier Saunders, 2012, vol 2, chap 50, p 1476. Motzer RJ, Mazumdar M, Bacik J, et al: Survival and prognostic stratification of 670 patients with advanced renal cell carcinoma. J CLIN ONCOL 1999;17:2530-2540.

2013 - 30 A 71-year-old man previously treated for CIS of the bladder has a positive fluorescence in-situ hybridization (FISH) urine test. Cytology, CT urogram, and cystoscopy are normal. The next step is: A. observation. B. repeat FISH test. C. random bladder biopsies. D. bilateral upper tract cytology. E. bilateral ureteroscopy.

C . The urinary FISH test identifies intranuclear chromosomal abnormalities that have been associated with bladder cancer. Specifically it detects aneuploidy for chromosome 3,7 and 17 and homozygous loss of chromosome 9p21. This test is currently FDA approved for the evaluation of microscopic hematuria and bladder cancer. In 2007 Yoder and colleagues reported that 35/56 (62.5%) patients with prior urothelial carcinoma who had a normal evaluation by cystoscopy and a positive FISH subsequently were detected to have recurrent disease. The appropriate workup of this patient population remains highly variable, however random bladder biopsies are considered the standard of care for patients with a positive urine cytology and negative cystoscopy and should be regarded as a minimum evaluation in this high risk patient. The yield of upper tract endoscopy and cytology in the setting of a normal CT urogram is low and should be discouraged as an initial diagnostic maneuver. Because the patient has a history of bladder cancer, the most likely site of recurrence is in the bladder. Repeat FISH will not add anything to the evaluation, whether positive or negative. Wood DP: Urothelial tumors of the bladder, Wein AJ, Kavoussi LR, Novick AC, Partin AW, Peters CA (eds): CAMPBELL-WALSH UROLOGY, ed 10. Philadelphia, Elsevier Saunders, 2012, vol 3, chap 80, pp 2327-2328. Yoder BJ, Skacel M, Hedgepeth R, et al: Reflex UroVysion testing of bladder cancer surveillance patients with equivocal or negative urine cytology: A prospective study with focus on the natural history of anticipatory positive findings. AM J CLIN PATHOL 2007;127:295-301.

2013 - 15 A 30-year-old man is diagnosed with stage 3 NSGCT. He undergoes radical orchiectomy and four cycles of BEP chemotherapy. His tumor markers have normalized. However, he has a 10 cm retroperitoneal mass and three 1 cm pulmonary masses (50% size reduction after chemotherapy). After his RPLND, the next step is: A. observation with serial imaging. B. PET scan with resection of lung masses if positive. C. resection of lung masses. D. resection of lung masses if retroperitoneum has active tumor. E. resection of lung masses if retroperitoneum has teratoma.

C . There is about 75% concordance between retroperitoneal pathology and pulmonary mass pathology, however, approximately 25% of cases will have discordant pathology (i.e., retroperitoneal fibrosis and active tumor or teratoma in the lung field). Therefore, post-chemotherapy thoracotomy yields important prognostic information and is curative in patients with resected teratoma and a subset of patents with viable tumor. PET scanning is a valuable decision making tool for retroperitoneal post-chemotherapy seminoma for residual masses greater than or equal to 3 cm. In this patient population, provided the PET scans are performed six weeks after the last chemotherapy cycle (decreased false positives), PET scans have a negative predictive value of 96% and a positive predictive value of 78% for active seminoma. This helps identify patients who merit additional treatment for post-chemotherapy seminoma retroperitoneal masses. PET scans usefulness, however, for the evaluation of supra-diaphragmatic, residual pulmonary nodules or mediastinal masses has not been extensively studied and recommendations for its use in this clinical situation have yet to be determined. Sheinfeld J, Bosl GJ: Surgery of testicular tumors, Wein AJ, Kavoussi LR, Novick AC, Partin AW, Peters CA (eds): CAMPBELL-WALSH UROLOGY, ed 10. Philadelphia, Elsevier Saunders, 2012, vol 1, chap 32, p 871.

2012 - 123 A five-year-old boy has precocious puberty. Scrotal ultrasound reveals a mass in the upper pole of the left testis. FSH and LH are normal prepubertal levels. Testosterone and urinary 17-ketosteroid levels are significantly elevated. The urinary pregnanetriol levels are normal. The next step is: A. glucocorticoid therapy. B. biopsy of the mass. C. enucleation of the mass. D. simple orchiectomy. E. radical orchiectomy.

C . This boy has the classic triad of Leydig cell tumors: precocious puberty, testis mass, and elevated 17-ketosteroid levels. Pituitary lesions may also cause precocious puberty except LH and FSH will be elevated in a prepubertal male. These tumors must be differentiated from hyperplastic nodules of CAH which occur when steroid replacement is inadequate. Although both entities have elevated urinary 17-ketosteroids, only CAH due to the 21-hydroxylase deficiency will cause an elevation in urinary pregnanetriol levels. Glucocorticoid replacement will cause regression of the hyperplastic nodules of CAH. Simple and radical orchiectomy is often performed but not preferred. These tumors are generally benign and preservation of testicular tissue with testis sparing surgery is recommended. Ritchey ML, Shamberger RC: Pediatric urologic oncology, in Wein AJ, Kavoussi LR, Novick AC, Partin AW, Peters CA (eds): CAMPBELL'S UROLOGY, ed 9. Philadelphia, Saunders Elsevier, 2007, vol 4, chap 130, p 3904.

2012 - 45 A one-year-old hypertensive boy has a large, fixed abdominal mass. The most likely diagnosis is: A. congenital mesoblastic nephroma. B. Wilms' tumor. C. neuroblastoma. D. pheochromocytoma. E. autosomal recessive polycystic kidney disease.

C . This boy likely has a neuroblastoma. 50% of these tumors present in children under the age of two. These tumors are usually large, hard, and fixed. Children will often have numerous other paraneoplastic syndromes. Catecholamine release from the neuroblastoma can result in symptoms that can mimic pheochromocytoma including paroxysmal hypertension, palpitations, sweating, and headaches. However, pheochromocytomas tend not to be large masses like this and present in older children. Wilms' tumor usually presents in children a few years older and the masses are more likely to be smooth and less fixed. Hypertension can also be seen but is less common. Congenital mesoblastic nephroma is possible but is usually seen in infants a few months of age and is the most common renal tumor in children less than six months of age. Autosomal recessive polycystic kidney disease can present at any age with a wide spectrum of symptoms. It involves both kidneys and you should be able to palpate bilateral masses. When it presents early in life, it is usually severe and associated with significant renal insufficiency. Ritchey ML, Shamberger RC: Pediatric urologic oncology, in Wein AJ, Kavoussi LR, Novick AC, Partin AW, Peters CA (eds): CAMPBELL'S UROLOGY, ed 9. Philadelphia, Saunders Elsevier, 2007, vol 4, chap 130, p 3871.

2014 - 134 During a routine sports physical, a 25-year-old man has a 2 cm, well-circumscribed, solid mass discovered in the lower pole of the left epididymis confirmed by ultrasound. The testis is palpably normal. The next step is: A. CT scan of the chest, abdomen, and pelvis. B. transscrotal excision of the mass. C. inguinal excision of the mass. D. left epididymectomy. E. left radical orchiectomy.

C . This lesion likely represents an adenomatoid tumor, however, one cannot rule-out sarcomas. Adenomatoid tumors are the most common tumor of the paratesticular tissues, accounting for approximately 30% of all paratesticular tumors. There has never been a documented case of metastasis. Most occur in individuals in their twenties or thirties. Tumors present as small, solid, asymptomatic masses generally found on routine examination, ranging from 0.5-4 cm in size. They are typically located in the anatomic distribution of the epididymis, testicular tunicae, and rarely, the spermatic cord. Most tumors will arise in or adjacent to the upper or lower pole of the epididymis, with a slightly higher incidence in the lower pole. The definitive treatment is surgical excision through an inguinal incision. Transscrotal incision is contraindicated because malignancy has not been ruled out. Fertility status, and/or semen analysis, and/or fertility preservation should be discussed with the patient prior to the procedure. Stephenson AJ, Gilligan TD: Neoplasms of the testis, Wein AJ, Kavoussi LR, Novick AC, Partin AW, Peters CA (eds): CAMPBELL-WALSH UROLOGY, ed 10. Philadelphia, Elsevier Saunders, 2012, vol 1, chap 31, p 869.

2012 - 119 A 76-year-old man has back pain. Seven years ago, he had a bilateral orchiectomy for T3NXM0 prostate cancer. MRI scan of the spine demonstrates a nonpathologic vertebral compression fracture. PSA is undetectable. The next step is: A. observation. B. bone scan. C. DEXA scan (bone densitometry). D. antiandrogen therapy. E. local radiotherapy.

C . This man, by virtue of his age and androgen deprivation therapy, is at increased risk for osteoporosis. The consequence of osteoporosis is an increase in bone fragility and a susceptibility to fracture. Androgen deprivation therapy (ADT) increases the risk of osteoporosis and is related to the duration of therapy. Compared to age-matched controls, men on ADT have 6.5% to 17.3% higher bone loss. Furthermore, the risk of non-pathologic fracture with ADT is 4% at five years and 20% at ten years. DEXA scan is the gold standard for diagnosis of osteoporosis, and is reported as compared to young adults (T-score) and age-matched (Z-score) controls. The treatment initially includes increasing physical activity on weight bearing joints and the addition of both Vitamin D and calcium. Since this patient has no clinical evidence of disease progression, there is no indication for additional treatment. MRI scan reveals only a nonpathologic fracture and PSA is undetectable; this combination obviates the need for a bone scan. Oefelein MG, Ricchuiti V, Conrad W, et al: Skeletal fracture associated with androgen suppression induced osteoporosis: the clinical incidence and risk factors for patients with prostate cancer. J UROL 2001;166:1724-1728. Nelson JB: Hormone therapy for prostate cancer, in Wein AJ, Kavoussi LR, Novick AC, Partin AW, Peters CA (eds): CAMPBELL'S UROLOGY, ed 9. Philadelphia, Saunders Elsevier, 2007, vol 3, chap 104, pp 3089-3090.

2012 - 95 A 79-year-old man with congestive heart failure has gross hematuria. CT urogram reveals a 6 mm distal right ureteral filling defect. Right ureteral washing for cytology are positive for high grade carcinoma. Serum creatinine of 1.9 mg/dl is stable. The next step is: A. retrograde pyelogram and ureteral washing in four months. B. ureteral stent placement and intravesical BCG. C. ureteroscopic biopsy and laser ablation. D. distal ureterectomy and reimplantation. E. nephroureterectomy.

C . This patient is elderly, has compromised medical and renal status and probably has a distal ureteral urothelial carcinoma. While optimal therapy in a younger, healthier patient would be distal ureterectomy and reimplantation, this patient would be well-served with endoscopic management. In the setting of subcentimeter lesions such as this one initial ureteroscopic ablation with followup ureteroscopy and surveillance is likely to be associated with equivalent control rates to distal ureterectomy and would avoid major abdominal surgery in this patient with heart failure. Sagalowsky AI, Jarrett TW: Management of urothelial tumors of the renal pelvis and ureter, in Wein AJ, Kavoussi LR, Novick AC, Partin AW, Peters CA (eds): CAMPBELL'S UROLOGY, ed 9. Philadelphia, Saunders Elsevier, 2007, vol 2, chap 49, pp 1673-1676.

2013 - 36 A man with castrate-resistant prostate cancer and bone metastases is on leuprolide acetate injections and intravenous zoledronic acid injections. He is asymptomatic. Zoledronic acid injections must be stopped if the patient develops: A. fever. B. fatigue. C. a tooth abscess. D. a pathologic fracture. E. severe osteoporosis.

C . Zoledronic acid and other bisphosphonates have become an important part of the management of patients with prostate cancer bone metastasis. These compounds reduce bone resorption by inhibiting osteoclastic activity and proliferation. In patients with progressive hormone refractory bone metastatic prostate cancer, zoledronic acid has been shown to reduce the incidence of skeletal events in a randomized prospective trial. Adverse events include fatigue, myalgias, fever, anemia and elevations in serum creatinine. Osteonecrosis of the mandibular bone is a severe complication of bisphosphonates usually associated with patients undergoing dental work or who have poor dentition or chronic dental disease. The bisphosphonates should be immediately discontinued in the setting of osteonecrosis or expected invasive dental procedures. Antonarakis ES, Carducci MA, Eisenberger MA: Treatment of castration-resistant prostate cancer, Wein AJ, Kavoussi LR, Novick AC, Partin AW, Peters CA (eds): CAMPBELL-WALSH UROLOGY, ed 10. Philadelphia, Elsevier Saunders, 2012, vol 3, chap 110, p 2954.

2012 - 36 A 69-year-old man with metastatic castrate resistant prostate cancer experiences symptomatic clinical progression following I.V. docetaxel and oral prednisone given every three weeks. In order to improve survival, the most appropriate next step is: A. mitoxantrone. B. sipuleucel-T. C. docetaxel weekly. D. cabazitaxel. E. ketoconazole.

D . A chemotherapeutic intervention proven to prolong survival in the setting of metastatic castrate resistant prostate cancer resistant to docetaxel is cabazitaxel. This regimen was FDA approved for this indication in 2010 and demonstrated a 30% reduction in the chance of dying of prostate cancer in a prospective phase III randomized trial. Mitoxantrone served as the control arm of the trial and has not been associated with a survival advantage in prostate cancer. Sipuleucel-T has been proven to extend survival in asymptomatic metastatic castrate resistant patients. Ketoconazole has not been proven to demonstrate a survival advantage in this setting and switching to weekly docetaxel has also not been associated with a survival advantage. Abiraterone, an androgen biosynthesis inhibitor has reported a survival difference as well in a phase III trial compared against placebo. de Bono JS, Oudard S, Ozguroglu M, et al; TROPIC Investigators: Prednisone plus cabazitaxel or mitoxantrone for metastatic castration-resistant prostate cancer progressing after docetaxel treatment: A randomized open-label trial. LANCET 2010;376:1147-1154.

2014 - 131 A 62-year-old man takes selenium and Vitamin E. He should be informed that his risk of: A. high-grade PIN will decrease. B. prostate cancer will decrease. C. type 2 diabetes will decrease. D. prostate cancer is unchanged. E. non-genitourinary cancers are increased.

D . A randomized, placebo-controlled trial Selenium and Vitamin E Cancer Prevention Trial [SELECT] of 35,533 men randomly assigned to four groups (selenium, Vitamin E, selenium/Vitamin E, and placebo) was performed in a double-blind fashion. Baseline eligibility included age 50 years or older (African American men) or 55 years or older (all other men), a serum prostate-specific antigen level of 4 ng/ml or less, and a digital rectal examination not suspicious for prostate cancer. Patients received oral selenium (200 _g/d from L-selenomethionine) and matched Vitamin E placebo, Vitamin E (400 IU/d of all race-alpha-tocopheryl acetate) and matched selenium placebo, selenium/Vitamin E, or placebo/placebo for a planned follow-up of minimum of seven years and a maximum of 12 years. There were no differences in prostate cancer risk between the four groups. However, there were statistically nonsignificant increased risks of prostate cancer in the Vitamin E group (P=.06) and type 2 diabetes mellitus in the selenium group (relative risk, 1.07; 99% CI, 0.94-1.22; P=.16) but not in the selenium/Vitamin E group. The conclusion of the study was that selenium or Vitamin E, alone or in combination at the doses and formulations used, did not prevent prostate cancer in this population of relatively healthy men. Non-genitourinary cancers were unchanged. Lippman SM, Klein EA, Goodman PJ, et al: Effect of selenium and Vitamin E on risk of prostate cancer and other cancers: The selenium and Vitamin E cancer prevention trial (SELECT). JAMA 2009;301:39-51.

2013 - 18 A 66-year-old man undergoes a radical nephrectomy with adrenalectomy for an 8 cm upper pole RCC within the kidney. There is a focus of non-contiguous, metastatic RCC in the adrenal gland. No lymph nodes were removed. According to the 2010 TNM AJCC classification, pathologic stage is: A. T1bN0Mx. B. T1bNxM1. C. T2aNxM0. D. T2aNxM1. E. T4NxM0.

D . According to the 2010 TNM AJCC staging, renal cancers greater than 7 cm and less than or equal to 10 cm are now categorized as pT2a. Lesions greater than 10 cm are pT2b. Adrenal gland involvement depends on whether there is contiguous involvement (T4) or non-contiguous involvement (M1). The M1 designation is true even if the adrenal gland is on the ipsilateral side as the nephrectomy. When there are no pathologic lymph nodes available, the pathologic staging is designated as NX. Edge SB, Byrd D, Compton C, Fritz A: AJCC Staging Manual, ed 7. New York, Springer-Verlag, 2010, chap 43, pp 479-490.

2014 - 101 A 33-year-old man with infertility has azoospermia. He has normal testes bilaterally and a serum FSH of 7 IU/l. Bilateral testis biopsies reveal maturation arrest and bilateral intratubular germ cell neoplasia. The next step is: A. serial ultrasound and tumor markers. B. repeat biopsy in six months. C. bilateral inguinal orchiectomy. D. XRT to both testes. E. systemic chemotherapy.

D . Due to the high association of subsequent development of overt carcinoma, testicular intratubular germ cell neoplasia may be treated by orchiectomy, radiation therapy, or chemotherapy. Radiation therapy is effective and may maintain Leydig cell function. In cases of bilateral disease, bilateral orchiectomy would be a drastic recommendation. Given the potential toxicities of chemotherapy compared to radiation, this approach would be reserved for contralateral CIS in the setting of metastasis from the primary tumor. Prior to treatment, testicular sperm extraction should be considered as some men with maturation arrest on diagnostic biopsy will have sperm found with a more extensive biopsy. Stephenson AJ, Gilligan TD: Neoplasms of the testis, Wein AJ, Kavoussi LR, Novick AC, Partin AW, Peters CA (eds): CAMPBELL-WALSH UROLOGY, ed 10. Philadelphia, Elsevier Saunders, 2012, vol 1, chap 31, pp 850-851.

2013 - 92 A 75-year-old man has severe bleeding from radiation cystitis requiring transfusion. Cystogram reveals no reflux. Previous therapeutic measures have failed including fulguration, clot evacuation, and irrigations with silver nitrate and 1% alum. The next step in management is: A. ileal loop urinary diversion. B. instillation of 10% formalin. C. instillation of 5% formaldehyde. D. instillation of 5% formalin. E. internal iliac artery embolization.

D . Formaldehyde is a 37% solution of formaldehyde gas dissolved in water and should not be used intravesically. Formalin solution is made up of 1-10% formaldehyde diluted with normal saline and has been given in bladder instillations to control hemorrhage from advanced bladder tumors or radiation cystitis. Formalin solution is exceedingly irritating to the bladder and, thus, requires general or regional anesthesia. Because a 10% formalin solution may cause fibrosis and obstruction of the ureteral orifices, formalin instillation should begin with a 1% solution and be repeated with a 5% and then a 10% solution, if necessary. Many people begin with a 5% solution if other measures (i.e., silver nitrate and 1% alum) have failed. A cystogram should be performed before instillation to rule out vesicoureteral reflux. If reflux is present, Fogarty catheters should be passed up both ureters, and the patient should be tilted into the head-up position to protect the upper tracts from the toxic effects of formalin. Selective internal iliac arterial embolization is more invasive and should be reserved for patients that fail formalin instillation. Smit SG, Heyns CF: Management of radiation cystitis. NAT REV UROL 2010;7:206-214.

2013 - 86 A 50-year-old man has a large right renal mass with tumor thrombus extending into the atrium. Under hypothermia and circulatory arrest, he undergoes nephrectomy with removal of the tumor thrombus. The most frequent significant complication is: A. hepatic dysfunction. B. pulmonary air embolus. C. central nervous system deficit. D. coagulopathy and hemorrhage. E. tumor emboli.

D . Hypothermia and circulatory arrest is the treatment of choice for a renal tumor with this level of cephalad extension. This technique has several potential complications such as CNS or hepatic damage yet the most common difficulty associated with this technique is hemorrhage associated with platelet and clotting factor dysfunction. Tumor emboli can occur but are relatively uncommon. Utilization of cardiopulmonary bypass limits the possibility of embolic events. Marshall FF, Dietrick DD, Baumgartner WA, Reitz BA: Surgical management of renal cell carcinoma with intracaval neoplasm extension above the hepatic veins. J UROL 1998;139:1166-1172. Kenney PA, Wotkowicz C, Libertino JA: Contemporary open surgery of the kidney, Wein AJ, Kavoussi LR, Novick AC, Partin AW, Peters CA (eds): CAMPBELL-WALSH UROLOGY, ed 10. Philadelphia, Elsevier Saunders, 2012, vol 2, chap 54, p 1624.

2013 - 90 In the management of advanced bladder cancer the substitution of carboplatin for cisplatin in a multidrug regimen has been shown to: A. not affect outcome. B. increase renal toxicity. C. improve survival. D. decrease response rate. E. increase duration of therapy.

D . In a meta-analysis of four randomized trials in 286 patients, Galsky and colleagues have concluded that the substitution of carboplatin for cisplatin resulted in a statistically significant (three fold decrease) in the probability of achieving a complete response and a significant decrease in the overall response rate. No significant effect on survival could be analyzed and in general the renal safety profile is improved with the use of carboplatin. The duration of therapy is not affected by the substitution of carboplatin. Galsky MD, Chen GJ, Oh WK, et al: Comparative effectiveness of cisplatin-based and carboplatin-based chemotherapy for treatment of advanced urothelial carcinoma. ANN ONCOL 2012;23:406-410.

2013 - 113 The risk factor associated with the highest chance for developing a secondary malignancy and/or cardiovascular disease in patients treated for testis cancer is: A. surveillance imaging. B. smoking. C. BEP chemotherapy. D. subdiaphragmatic and mediastinal radiation therapy. E. cisplatin, vinblastine, bleomycin (PVB) chemotherapy.

D . In a multivariate analysis, combined subdiaphragmatic and mediastinal radiation has a Hazard's Ratio (HR) of 3.7 for secondary malignancy and/or cardiovascular disease; chemotherapy also increased the risk (HR 1.9), but there is no difference between the chemotherapy regimens. Smoking increased risk 1.7 fold. There is no increased risk of secondary malignancy with surgery. Surveillance imaging is a risk factor for developing secondary malignancy, but the magnitude of the risk is much lower than radiation therapy. Haugnes HS, Wethal T, Aass N, et al: Cardiovascular risk factors and morbidity in long-term survivors of testicular cancer: A 20-year follow-up study. J CLIN ONCOL 2010;28:4649-4657. Abouassaly R, Fossa SD, Giwercman A, et al: Sequale of treatment of testis cancer. EUR UROL 2011;60:516-526. Fung C, Vaughn DJ: Complication associated with chemotherapy in testicular cancer. NATURE REVIEWS UROL 2011;8:213-222.

2011 - 9 A 47-year-old man has palpable right inguinal adenopathy following partial penectomy for a 4 cm T2 squamous cell cancer. Needle biopsy of a right inguinal lymph node reveals metastatic cancer. The pelvic lymph nodes are radiologically normal. The next step is: A. antibiotic therapy and reexamination. B. pelvic node dissection. C. right superficial inguinal node dissection. D. right superficial and deep, left superficial inguinal node dissection. E. bilateral superficial and deep inguinal node dissection.

D . Men with invasive penile cancer are at high risk of inguinal metastasis. Those men who present with palpable inguinal lymph nodes often have an inflammatory or infectious etiology due to poor hygiene. In these patients, two approaches can be employed. Patients can be treated with a two to four week course of antibiotic therapy to assess for resolution of lymphadenopathy. Alternatively, fine needle aspirate of suspicious nodes can be performed at presentation. If positive, this removes the need for delayed therapy due to antibiotics. Men with palpable nodes proven positive for metastatic disease should undergo superficial and deep inguinal node dissection as those with limited nodal disease are found to have up to an 80% five year disease-free survival with complete resection of nodal disease. Owing to the high rates of cure achieved with aggressive resection in limited nodal disease, many have advocated early dissection in men with invasive penile cancer and no palpable lymphadenopathy. In these men, dissection can be limited to nodes superficial to the fascia lata unless positive on evaluation. In men with palpable disease on one side, contralateral superficial dissection is mandatory owing to the high rate of lymphatic cross-over. In these cases, contralateral metastasis is noted in 50% of patients. Pettaway CA, Lynch DF Jr, Davis JW: Tumors of the penis, in Wein AJ, Kavoussi LR, Novick AC, Partin AW, Peters CA (eds): CAMPBELL'S UROLOGY, ed 9. Philadelphia, Saunders Elsevier, 2007, vol 1, chap 31, pp 983-984.

2014 - 137 A 67-year-old man underwent radical prostatectomy with pelvic lymph node dissection following three months of LH-RH agonist therapy. The pathologist will not be able to accurately describe: A. tumor volume. B. tumor stage. C. nodal status. D. Gleason score. E. surgical margins.

D . Several series have demonstrated that administration of hormonal deprivation therapy prior to radical prostatectomy does not lower the risk of biochemical relapse. Although tumor volume, stage, nodal status, and margin status appear improved in men receiving neoadjuvant hormonal ablation, the risk of biochemical relapse is equivalent to those men who do not receive any neoadjuvant therapy. Gleason scoring can be quite difficult in men who have received hormonal deprivation due to the noted treatment effect in the tissues. Among these men, Gleason score can appear artificially higher and it is recommended that Gleason score not be assigned. Epstein JI: Pathology of prostatic neoplasia, Wein AJ, Kavoussi LR, Novick AC, Partin AW, Peters CA (eds): CAMPBELL-WALSH UROLOGY, ed 10. Philadelphia, Elsevier Saunders, 2012, vol 3, chap 96, p 2732.

2012 - 27 A 68-year-old man undergoes a partial penectomy for a 4 cm squamous cell carcinoma with lymphovascular invasion and involvement of the subepithelial connective tissue. Physical exam reveals a 1.5 cm fixed, right inguinal mass. CT scans of the abdomen and pelvis are normal. His pathologic tumor stage (p) and clinical lymph node stage (c) are: A. pTa cN1. B. pT1a cN1. C. pT1b cN2. D. pT1b cN3. E. pT2 cN3.

D . The 2010 AJCC staging for penile cancer made several significant changes. This patient has a pT1 tumor because of his subepithelial connective tissue involvement. Those patients with low grade tumors and without lymphovascular invasion are pT1a. This patient, however, has lymphovascular invasion and as a result is a pT1b. Patients with lymphovascular invasion are in fact at higher risk for metastatic disease. For the first time, nodal staging is divided into both clinical and pathologic staging schemes. With a palpable, fixed nodal mass, regardless of the size or unilateral/bilateral involvement, the clinical lymph node status is cN3. Edge, SB, Byrd D, Compton C, Fritz A: AJCC Staging Manual, ed 7. New York, Springer-Verlag, 2010, chap 40, pp 449-450.

2014 - 27 Screening for RCC in patients with ESRD should be reserved for patients: A. with significant comorbidities. B. initiating hemodialysis. C. with a history of obesity and tobacco abuse. D. who have undergone at least three years of dialysis. E. with autosomal dominant polycystic kidney disease.

D . The relatively low incidence of RCC in the general population and the potential risk of identifying clinically insignificant lesions argue against widespread screening. However, several well-defined target populations who are at increased risk of RCC may be suitable for screening efforts. The relative risk of RCC in patients with ESRD has been estimated to be 5-20 times higher than that in the general population. The majority of patients with ESRD will develop acquired renal cystic disease (ARCD) and some of these patients will develop RCC. Cystic disease is associated with time of maintenance dialysis; increasing with duration of therapy. In renal failure patients, a reasonable approach to screening for patients with ESRD without other major comorbidities, is to delay screening until the third year on dialysis. Those with significant comorbidities, and thereby limited life expectancies, are not felt to benefit from screening and thereby should not be screened. Patients undergoing maintenance hemodialysis were previously thought to have a higher incidence of ARCD and thereby a higher risk of renal carcinoma than patients on peritoneal dialysis. However, recent studies have demonstrated similar rates of ARCD in both dialysis subgroups, and thereby suggests both forms of maintenance dialysis would increase the risk of developing renal carcinoma. Obesity and tobacco abuse have been shown to be risk factors for RCC, but these conditions themselves do not necessarily warrant screening efforts in patients with renal dysfunction. Screening for RCC in autosomal dominant polycystic kidney disease (ADPKD) was previously recommended. However, recent studies suggest no significantly increased risk of RCC in ADPKD and imaging is extremely difficult in this population related to the altered intrarenal architecture. Taken together, these considerations suggest that screening for RCC in patients with ADPKD should not be pursued. Campbell SC, Lane BR: Malignant renal tumors, Wein AJ, Kavoussi LR, Novick AC, Partin AW, Peters CA (eds): CAMPBELL-WALSH UROLOGY, ed 10. Philadelphia, Elsevier Saunders, 2012, vol 2, chap 49, pp 1438-1439.

2012 - 128 A 62-year-old man has a radical prostatectomy for prostate cancer. Histology reveals a Gleason 9, pT3aN1Mx cancer with negative surgical margins. His post-prostatectomy PSA is < 0.1 ng/ml. To minimize his risk of relapse, the next step is: A. adjuvant docetaxel. B. external beam XRT. C. LH-RH agonist therapy for six months. D. lifelong LH-RH agonist therapy. E. LH-RH agonist therapy for six months and external beam XRT.

D . The treatment of men with isolated lymph node metastasis at the time of prostate cancer surgery has been controversial. The use of adjuvant XRT for adverse pathologic characteristics at the time of prostatectomy has been recently evaluated in two large randomized clinical trials. However, patients with lymph node metastasis, such as patients here, were not included in this trial, and it is generally felt that such individuals are at risk of systemic rather than local recurrence. ECOG 3807 evaluated the use of immediate hormonal ablation versus observation in men with isolated lymph node metastases noted on final pathology after radical prostatectomy. The patients treated with immediate therapy had improved overall and cancer-specific survival relative to men undergoing initial observation. No trial has been performed showing an advantage of any chemotherapy treatment in the adjuvant setting. The advantage of short-term androgen deprivation therapy in this setting has not been established. Nelson JB: Hormone therapy for prostate cancer, in Wein AJ, Kavoussi LR, Novick AC, Partin AW, Peters CA (eds): CAMPBELL'S UROLOGY, ed 9. Philadelphia, Saunders Elsevier, 2007, vol 3, chap 104, pp 3095-3098. Meng MV, Carroll PR: Treatment of locally advanced prostate cancer, in Wein AJ, Kavoussi LR, Novick AC, Partin AW, Peters CA (eds): CAMPBELL'S UROLOGY, ed 9. Philadelphia, Saunders Elsevier, 2007, vol 3, chap 102, pp 3056-3057. Catalona WJ, Han M: Definitive therapy for localized prostate cancer - An overview, in Wein AJ, Kavoussi LR, Novick AC, Partin AW, Peters CA (eds): CAMPBELL'S UROLOGY, ed 9. Philadelphia, Saunders Elsevier, 2007, vol 3, chap 95, p 2938.

2013 - 81 A 14-year-old boy has a painless mass above the right testis. Radical orchiectomy is performed. Pathology reveals a completely resected paratesticular rhabdomyosarcoma. His abdominal CT scan is normal. The next step is: A. repeat CT scan in three months. B. adjuvant chemotherapy with doxorubicin and alkylating agents. C. retroperitoneal radiation. D. ipsilateral RPLND. E. bilateral RPLND.

D . There is a significant rate of false negative imaging of the retroperitoneum on CT scan and microscopic disease may be present in patients with paratesticular rhabdomyosarcoma. Boys ten years of age or younger can be treated with chemotherapy alone. Boys over ten years of age should undergo ipsilateral RPLND since 50% will have microscopic disease. Bilateral RPLND is more morbid and does not improve survival over unilateral surgery. If the lymph nodes are positive for metastasis, the patient will require radiation therapy to the retroperitoneum in addition to chemotherapy. Ritchey ML, Shamberger RC: Pediatric urologic oncology, Wein AJ, Kavoussi LR, Novick AC, Partin AW, Peters CA (eds): CAMPBELL-WALSH UROLOGY, ed 10. Philadelphia, Elsevier Saunders, 2012, vol 4, chap 137, p 3709.

2014 - 89 A healthy 50-year-old man with gross hematuria has a peripherally located 4 cm solid mass in a solitary kidney. The serum creatinine is 1.2 mg/dl. CT scan demonstrates a 1 cm renal vein tumor thrombus. There is no evidence of metastasis. The next step is: A. angiographic embolization. B. laparoscopic cryoablation. C. partial nephrectomy. D. radical nephrectomy. E. interleukin-2.

D . Under usual circumstances, a peripherally located tumor in a solitary kidney is best managed by partial nephrectomy, both for potential cure and to prevent the need for dialysis. However, the presence of a venous thrombus makes complete tumor excision less likely and is associated with a high likelihood of tumor recurrence and poor prognosis after partial nephrectomy. Given the patient's good health, long life expectancy, and episode of gross hematuria, observation is likely to result in further tumor progression, more bleeding or other local symptoms which may require intervention, and potentially metastatic disease. Laparoscopic cryoablation is best used for smaller tumors and will not control the tumor thrombus. Systemic immunotherapy rarely results in a response in the primary tumor and is unlikely to be curative. Thus, the best choice is radical nephrectomy which will result in the need for dialysis. A prolonged disease-free interval would make the patient eligible for subsequent renal transplantation. Selective angioinfarction of the tumor is another alternative which could be used for palliation of the bleeding, but will not be curative. Angermeier KW, Novick AC, Streem SB, et al: Nephron-sparing surgery for renal cell carcinoma with venous involvement. J UROL 1990;144:1352-1355. Campbell SC, Lane BR: Malignant renal tumors, Wein AJ, Kavoussi LR, Novick AC, Partin AW, Peters CA (eds): CAMPBELL-WALSH UROLOGY, ed 10. Philadelphia, Elsevier Saunders, 2012, vol 2, chap 49, p 1445.

2014 - 110 A 67-year-old man with a history of six weekly BCG treatments undergoes four cycles of M-VAC for a muscle invasive urothelial cell carcinoma (T3bNXMO) at the bladder neck. Following chemotherapy, there is no visible tumor although the urine cytology is positive. The next step is: A. cystoscopy in three months. B. repeat six weeks of BCG. C. maintenance BCG. D. XRT. E. radical cystoprostatectomy.

E . Among patients with invasive bladder cancer, M-VAC chemotherapy will result in a complete pathologic response in 20% of patients prior to cystectomy. However, between 20-30% of patients with a clinical response will harbor residual invasive disease at the time of cystectomy. Accordingly, following chemotherapy this patient should undergo a cystoprostatectomy. There is no role for additional intravesical therapy in this patient. In addition, it would be inappropriate to place him on a surveillance protocol. HB Grossman, RB Natale, CM Tangen, et al: Neoadjuvant chemotherapy plus cystectomy compared with cystectomy alone for locally advanced bladder cancer. NEJM 2003;349:859-866. Lerner SP, Sternberg CN: Management of metastatic and invasive bladder cancer, Wein AJ, Kavoussi LR, Novick AC, Partin AW, Peters CA (eds): CAMPBELL-WALSH UROLOGY, ed 10. Philadelphia, Elsevier Saunders, 2012, vol 3, chap 82, p 2361.

2014 - 36 A 70-year-old man is undergoing radical cystectomy and continent orthotopic urinary diversion for muscle-invasive high-grade urothelial carcinoma. During lymphadenectomy, a suspicious, firm 1.5 cm positive external iliac lymph node is confirmed to be positive for metastasis. The next step is: A. abort surgery and treat with chemotherapy and XRT. B. abort surgery and treat with chemotherapy followed by cystectomy. C. perform lymphadenectomy and treat with chemotherapy and XRT. D. complete surgery but perform an ileal conduit urinary diversion. E. complete surgery as planned.

E . At the time of radical cystectomy, suspicious lymph nodes can be encountered. Approximately 25% of patients will ultimately have positive lymph nodes at the time of radical cystectomy. If the suspicious node(s) can be safely resected and the volume of suspicious lymph nodes is limited, it is reasonable to continue the cystectomy and continent urinary diversion. The patient will benefit from the local control of the lymphadenectomy and cystectomy. There is no evidence that stopping the surgery to treat with neoadjuvant chemotherapy or chemotherapy and radiation therapy is superior to completing the cystectomy. Positive lymph nodes are not a contraindication to continent urinary diversion. Lerner SP, Sternberg CN: Management of metastatic and invasive bladder cancer, Wein AJ, Kavoussi LR, Novick AC, Partin AW, Peters CA (eds): CAMPBELL-WALSH UROLOGY, ed 10. Philadelphia, Elsevier Saunders, 2012, vol 3, chap 82, p 2360.

2013 - 83 A 68-year-old woman has nocturia x3, persistent suprapubic pain, urgency and daytime frequency after her third BCG instillation for recurrent stage Ta bladder cancer. A urinalysis reveals 5-10 RBC and 10-20 WBC/hpf. A urine culture is negative. The next step is: A. space remaining treatments two weeks apart. B. ciprofloxacin. C. decrease weekly intravesical dose of BCG by 50 percent. D. isoniazid therapy with BCG treatments. E. oxybutynin.

E . BCG cystitis is a common side effect of BCG therapy. A possible bacterial infection should also always be considered. Quinolone antibiotics are not indicated with a negative urine culture and may have a negative effect on BCG therapy as it is partially tuberculocidal. Similarly, antitubercular medications may abrogate the effectiveness of BCG and add the risk of hepatotoxicity. A decrease in dose intensity may hamper therapeutic efficacy. The patient's symptoms are mild to moderate and are best treated symptomatically with an anti-spasmodic agent during the course of therapy. Jones JS, Larchian WA: Non-muscle-invasive bladder cancer (Ta, T1, and CIS), Wein AJ, Kavoussi LR, Novick AC, Partin AW, Peters CA (eds): CAMPBELL-WALSH UROLOGY, ed 10. Philadelphia, Elsevier Saunders, 2012, vol 3, chap 81, p 2345. van der Meijden AD PM, Klingeren BV, Steerenberg PA: The possible influence of antibiotics on results of bacillus Calmette-Guerin intravesical therapy for superficial bladder cancer. J UROL 1992;147:596-600.

2014 - 125 A 61-year-old man has an International Prostate Symptom Score (IPSS) score of 18, a 40 gm benign feeling prostate on DRE, and a PSA of 3.2 ng/ml. He was started on dutasteride and the LUTS improved markedly during the first year of treatment. PSA data on follow-up is: 12 months, 1.2 ng/ml; 18 months, 0.9ng/ml; 24 months, 1.4 ng/ml; and 36 months, 1.9 ng/ml. The next step is: A. PCA3. B. free and total PSA in six months. C. PSA in one year. D. quinolone antibiotic followed by repeat PSA in six weeks. E. TRUS and prostate biopsy.

E . Currently available 5-alpha-reductase inhibitors (finasteride and dutasteride) lower serum PSA in patients with and without prostate cancer. The doubling rule has been used to compensate for PSA reductions in patients on these medications. With growing experience and data from long-term clinical trials using these drugs (e.g. PCPT and PLESS), the effect on PSA and prostate cancer detection has been more clearly defined. Although the doubling rule does accurately reflect the population as a whole, given the wide intra and interindividual variability of serum PSA changes while on these medications, it is relatively inaccurate in the individual patient. While treatment of patients with either finasteride or dutasteride results in a -50% median change in the group as a whole, the 5th to 95th percentile range is -81-20%. Recent analysis of long-term data regarding the change in PSA in men receiving 5-alpha-reductase inhibitors, suggests that in the first year the doubling rule overestimates PSA and can systematically lead to an increased likelihood of prostate biopsy. Beyond 24 months (with further decline in PSA), the doubling rule may underestimate PSA and result in fewer biopsies. As most patients receiving these drugs attain a nadir serum PSA, a sustained increase from nadir can be used to prompt biopsy the same way as it would in untreated patients. Using a 0.3 ng/ml increase from nadir as a trigger for biopsy maintains a sensitivity and specificity similar to an absolute value of 4.0 ng/ml in untreated patients. Free and total PSA ratio should remain unaffected by 5-alpha-reductase inhibitor use. In the absence of infection, there is no indication for use of antibiotics. PCA3 is only FDA-approved for patients with a prior history of a negative biopsy. Marks LS, Andriole GL, Fitzpatrick JM, et al: The interpretation of serum prostate specific antigen in men receiving 5-alpha-reductase inhibitors: A review and clinical recommendations. J UROL 2006;176:868-874.

2014 - 98 Deletions of the short arm of chromosome 3 occur most frequently in association with: A. papillary RCC. B. angiomyolipoma. C. renal medullary carcinoma. D. oncocytoma. E. clear cell RCC.

E . Deletions of the short arm of chromosome 3 are characteristic of clear cell (conventional type) RCCs, which are seen in sporadic cases of renal cell carcinoma and those associated with von Hippel-Lindau disease. The short arm of chromosome 3 is the location of the VHL gene. Type 1 papillary renal cancers are associated with mutations of the c-met proto-oncogene located on chromosome 7. Type II papillary renal cancers are associated with mutations of the fumarate hydratase gene located on chromosome 1. Oncocytoma can be seen in mutations of the BHD1 gene located on the short arm of chromosome 17. Campbell SC, Lane BR: Malignant renal tumors, Wein AJ, Kavoussi LR, Novick AC, Partin AW, Peters CA (eds): CAMPBELL-WALSH UROLOGY, ed 10. Philadelphia, Elsevier Saunders, 2012, vol 2, chap 49, p 1425.

2014 - 95 Left inguinal orchiectomy is performed on a 24-year-old man for an embryonal cell carcinoma. AFP is elevated but beta-hCG is normal. On chest x-ray, there is a 5 cm mass in the right lung and an abdominal CT scan shows a 2 cm periaortic adenopathy. After four cycles of platinum-based chemotherapy, AFP has returned to normal, and an abdominal CT scan shows resolution of the retroperitoneal adenopathy. On chest x-ray, the lung mass is still present but has decreased to 3 cm in size. The next step is: A. retroperitoneal node dissection. B. salvage chemotherapy with ifosfamide. C. continue platinum chemotherapy for two more cycles. D. needle biopsy of the lung mass. E. pulmonary wedge resection.

E . Further chemotherapy is not indicated without histologic evidence of residual disease. Although assessment of the retroperitoneal nodes must be considered, removal of the lung mass by thoracotomy is indicated first. Needle biopsy would be an insufficient sampling method and if this is teratoma, excision of the lung mass would be therapeutic. Bosl GS, Motzer RJ: Testicular germ cell cancer. NEJM 1997;337:242-253.

2011 - 15 A 24-year-old man elects to undergo a modified right template RPLND following right radical orchiectomy for stage I NSGCT. The left limit of the dissection should be: A. the medial edge of vena cava. B. the medial edge of the aorta. C. the mid-aorta. D. the lateral edge of the aorta. E. the medial edge of the left ureter.

E . Lymphatic spread of testicular cancer to the contralateral retroperitoneum is rare with left-sided tumors, but more common with right sided tumors. To this end, the contralateral margin differs for left- and right-sided modified RPLND templates. For left sided dissections, the right margin is the lateral edge of the IVC, primarily to ensure collection of the interaortacaval lymph nodes. On the right side, the dissection should be carried out further, optimally to the left ureter, as occasionally there will be involvement of the para-aortic lymph nodes in these patients. Although some authors have suggested that bilateral modified RPLND should be performed in all patients with right-sided tumors, the additional dissection down the left common iliac artery has not been shown to be of any additional benefit. Richie JP, Steele GS: Neoplasms of the testis, in Wein AJ, Kavoussi LR, Novick AC, Partin AW, Peters CA (eds): CAMPBELL'S UROLOGY, ed 9. Philadelphia, Saunders Elsevier, 2007, vol 1, chap 29, pp 915-916.

2014 - 40 A 45-year-old man undergoes a partial penectomy for a 3 cm, grade 3, squamous cell carcinoma of the penis. Pathology reveals invasion of the corpus cavernosum and negative margins. On exam, he has matted firm 6 cm right inguinal lymph nodes and a CT scan of the chest, abdomen, and pelvis reveals right sided inguinal adenopathy but no other metastases. The next step is: A. percutaneous biopsy of the inguinal lymph nodes. B. right inguinal lymph node dissection. C. bilateral inguinal lymph node dissection. D. bilateral inguinal and pelvic lymph node dissection. E. neoadjuvant chemotherapy.

E . Neoadjuvant platinum-based chemotherapy is indicated for patients with difficult to resect matted or fixed inguinal lymph nodes from metastatic penile cancer. The neoadjuvant chemotherapy should reduce the size of the nodes and enhance their resectability. Following neoadjuvant chemotherapy, the patient should have a bilateral inguinal and pelvic lymphadenectomy. A percutaneous biopsy is not necessary in this setting. Pettaway CA, Lance RS, Davis JW: Tumors of the penis, Wein AJ, Kavoussi LR, Novick AC, Partin AW, Peters CA (eds): CAMPBELL-WALSH UROLOGY, ed 10. Philadelphia, Elsevier Saunders, 2012, vol 1, chap 34, p 926.

2014 - 143 A 65-year-old man has a partial penectomy for a high-grade pT2 lesion. He has no palpable adenopathy in the inguinal region. Metastatic workup is negative. The next step in management is: A. serial physical examination of the inguinal nodes. B. serial imaging with CT scan. C. sentinel inguinal node biopsy. D. delayed inguinal lymphadenectomy if nodes become palpable. E. immediate bilateral inguinal lymphadenectomy.

E . Patients with high-grade T2 cancer are at a significant risk of occult regional disease or of development of this disease. Studies have shown that patients who have an immediate bilateral inguinal lymphadenectomy have an increased survival than if they develop palpable disease during follow-up. Although this patient currently has no evidence of adenopathy, he has a 59% risk of developing involved lymph nodes. Sentinel lymph node biopsy is gaining popularity, but still lacks sufficient sensitivity and specificity. Pettaway CA, Lance RS, Davis JW: Tumors of the penis, Wein, AJ, Kavoussi LR, Novick AC, Partin AW, Peters CA (eds): CAMPBELL-WALSH UROLOGY, ed 10. Philadelphia, Elsevier Saunders, 2012, vol 1, chap 34, p 916.

2013 - 88 Patients with von-Hippel Lindau disease most frequently have: A. renal angiomyolipoma. B. cafe-au-lait spots. C. glioblastomas. D. thyroid carcinoma. E. retinal angiomas.

E . Patients with von-Hippel Lindau disease may have hemangioblastomas of the cerebellum, renal cell carcinomas, and cystadenomas of the epididymis. The diagnosis, however, can often be made most easily with inspection of the retina with identification of angiomas. Renal angiomyolipomas are commonly seen in tuberous sclerosis complex. Thyroid carcinoma can be seen more commonly in patients with multiple endocrine neoplasia syndrome. Cafe-au-lait spots are pathognomonic of neurofibromatosis. Neumann HP, Berger DP, Sigmund G, Blum U, et al: Pheochromocytomas, multiple endocrine neoplasia type 2, and von Hippel-Lindau disease. NEJM 1993;329(21):1531-1538. Campbell SC, Lane BR: Malignant renal tumors, Wein AJ, Kavoussi LR, Novick AC, Partin AW, Peters CA (eds): CAMPBELL-WALSH UROLOGY, ed 10. Philadelphia, Elsevier Saunders, 2012, vol 2, chap 49, p 1423.

2014 - 149 A 68-year-old active man with metastatic castration-resistant prostate cancer receives six cycles of docetaxel plus prednisone. Bone scan reveals several new rib lesions. To improve the likelihood of overall survival, the next treatment step is prednisone and: A. continued docetaxel. B. zoledronic acid. C. ketoconazole. D. mitoxantrone. E. cabazitaxel.

E . Progression while on docetaxel-chemotherapy defines failure of the docetaxel. Cabazitaxel is a novel tubulin-binding taxane drug with anti-tumor activity in docetaxel-resistant cancers. In a large Phase 3 trial, the efficacy and safety of cabazitaxel plus prednisone was compared to that of mitoxantrone plus prednisone. There was a 30% reduction in the risk of death with the cabazitaxel (15.1 months versus 12.7 months). None of the other agents listed have been shown to improve survival in men with castration-resistant prostate cancer. Two additional agents recently approved by the FDA which are abiraterone and enzalutamide, have been shown to also improve survival in this patient population. None of the other regimens have demonstrated a survival difference in this population of patients. de Bono JS, Oudard S, Ozguroglu M, et al: Prednisone plus cabazitaxel or mitoxantrone for metastatic castration-resistant prostate cancer progressing after docetaxel treatment: A randomised open-label trial. LANCET 2010;376:1147-1154. Cookson MS, Kibel AS, Dahm P, et al: Castration-resistant prostate cancer: AUA guideline. CASTRATION-RESISTANT PROSTATE CANCER. American Urological Association Education and Research, Inc, 2013. http://www.auanet.org/education/guidelines/castration-resistant-prostate-cancer.cfm

2012 - 83 The best predictor of long-term recurrence-free status in a patient with non-invasive bladder cancer is: A. tumor size. B. Rb mutation. C. tetraploid flow cytometry. D. negative random biopsies. E. negative cystoscopy three months after a TURBT.

E . Rb mutations (Retinoblastoma Protein) are found in approximately 30% of bladder tumors and are correlated with a higher stage disease at time of diagnosis and decreased patient survival. Flow cytometry has not been found to be of more clinical value than conventional cytology in determining tumor recurrence. Low grade tumors are usually diploid, bladder tumors with triploid to tetraploid will be found to have unfavorable pathologic characteristics and a poorer prognosis. Interesting patients with pure tetraploid expression have a more favorable prognosis than those of patients with triploid to tetraploid tumors but worse than diploid tumors. Both tumor size and negative random biopsies have a dramatic impact on progression rate but less on recurrence rate. In patients with a well or moderated differentiated non-invasive bladder cancer, the factor which best predicts the absence of tumor recurrence is a post TURBT negative cystoscopy at three months. If no tumor is present at three months, there is an 80% chance of no further tumor recurrence. Jones JS, Campbell SC: Non-muscle-invasive bladder cancer (Ta, T1, and CIS), in Wein AJ, Kavoussi LR, Novick AC, Partin AW, Peters CA (eds): CAMPBELL'S UROLOGY, ed 9. Philadelphia, Saunders Elsevier, 2007, vol 3, chap 76, p 2463.

2014 - 83 A 55-year-old man has a transrectal biopsy which reveals small cell carcinoma. Metastatic workup is negative. The next step is: A. radical prostatectomy. B. LH-RH agonist. C. cystoprostatectomy. D. external beam radiation therapy (EBRT). E. chemotherapy.

E . Small-cell carcinoma of the prostate carries a very poor prognosis. In most patients, the course is rapidly fatal. Most patients present with rapid onset of symptoms indistinguishable from other causes of bladder outlet obstruction. Systemic constitutional symptoms are reported in about 10% of cases. Chemotherapy in combination with surgery or radiation appears to be the most important component of management. Radiation is ineffective as is surgery alone, since most patients have metastatic disease. These tumors are generally not hormonally sensitive. Abbas F, Civantos F, Benedett P, et al: Small cell carcinoma of the bladder and prostate. J UROL 1995;46:30. Antonarakis ES, Carducci MA, Eisenberger MA: Treatment of castration-resistant prostate cancer, Wein AJ, Kavoussi LR, Novick AC, Partin AW, Peters CA (eds): CAMPBELL-WALSH UROLOGY, ed 10. Philadelphia, Elsevier Saunders, 2012, vol 3, chap 110, pp 2961-2962.

2013 - 140 A 68-year-old diabetic man has a 1 cm left proximal ureteral non-invasive, high grade urothelial carcinoma. His serum creatinine is 0.9 mg/dl. Abdominal CT scan is otherwise normal. The next step is: A. ureteroscopic tumor ablation, stent placement, and intravesical BCG instillation. B. percutaneous resection and antegrade BCG. C. left partial ureterectomy. D. left ureterectomy with Boari flap. E. left radical nephroureterectomy.

E . Standard therapy for patients with upper tract urothelial carcinoma involving the proximal ureter is nephroureterectomy. Endoscopic treatment of patients with upper tract urothelial carcinoma is generally recommended in those patients with a solitary kidney, bilateral disease, renal dysfunction, or significant intercurrent illness that precludes a major abdominal procedure. Endoscopic management may also be appropriate in selected patients with small, low-grade lesions in the presence of a normal contralateral kidney. However, most series suggest that recurrence is likely even with frequent reinspection and that progression to invasive disease occurs in a significant number of patients, depending on the stage and grade of the initial tumor. Open ureteral resection risks tumor spillage as well as recurrence. This patient has a normal contralateral right kidney. A recent large, multi-institutional examination of nephrouterectomy and ipsilateral lymph node dissection did not reveal any survival advantage for patients treated with node dissection with earlier stage upper tract malignancy as is the case with this patient. However, in patients with T2-T4 primary tumors, pathologic N0 patients did have a longer cancer-specific survival than pathologic Nx patients. Roscigno M, Shariat SF, Margulis V, et al: Impact of lymph node dissection on cancer specific survival in patients with upper tract urothelial carcinoma treated with radical nephroureterectomy. J UROL 2009;181:2482-2489. Sagalowsky AI, Jarrett TW, Flanigan RC: Urothelial tumors of the upper urinary tract and ureter, Wein AJ, Kavoussi LR, Novick AC, Partin AW, Peters CA (eds): CAMPBELL-WALSH UROLOGY, ed 10. Philadelphia, Elsevier Saunders, 2012, vol 2, chap 53, p 1516.

2013 - 146 A 63-year-old woman with metastatic clear cell RCC and a poor performance status has a serum calcium of 11 mg/dl and a hemoglobin of 8 g/dl. The most appropriate treatment is: A. interferon. B. bevacizumab. C. sunitinib. D. sorafenib. E. temsirolimus.

E . Temsirolimus is an inhibitor of the mammalian target of rapamycin (mTOR) kinase. This is a component of intracellular signaling pathways involved in growth/proliferation of cells. The medication suppresses angiogenesis and is given as a weekly I.V. infusion. Patients with three or more poor risk factors respond better to mTOR inhibitors than other currently used medications. Risk factors include: serum LDH > 1.5 times upper limit of normal, hemoglobin below lower limit of normal, serum calcium level of more than 10 mg/dl, time from initial diagnosis of RCC to randomization of less than one year, Karnofsky performance of 60 or 70, or metastases in multiple organs. This patient is considered poor risk and patients who received this regimen were 27% more likely to survive when compared to patients receiving interferon alpha alone. This medication is FDA-approved for advanced RCC. In addition bevacizumab, sunitinib and sorafenib have not been proven to have a survival benefit in this high risk subset of patients. Interferon has not demonstrated a consistent survival advantage in the management of metastatic RCC. Hudes G, Carducci M, Tomczak P, et al: Temsirolimus, interferon alpha, or both for advanced renal-cell carcinoma. NEJM 2007;356:2271-2281.

2013 - 107 A 59-year-old man undergoes TRUS biopsy with a PSA of 4.5 ng/ml and a normal DRE. The biopsy reveals BPH and nine months later his PSA has risen to 5 ng/ml and he undergoes a PCA3 urine test. The PCA3 score is 99 (low risk < 35). The next step is: A. 5-alpha reductase inhibitor. B. one month of ciprofloxacin and repeat PCA3. C. free to total PSA ratio. D. prostate MRI. E. repeat TRUS biopsy.

E . The PCA3 urine test is designed to detect the mRNA signal from the DD3 gene which is specific to prostate cancer. This test has now been validated in multiple prospective studies the largest of which was the REDUCE clinical trial. In this study PCA3 scores were measured in 1,072 subjects with a negative prior TRUS biopsy and the results were more closely correlated to the results of the second biopsy than PSA alone or free to total PSA ratio. The higher the level of the PCA3 score the higher the risk of a positive biopsy and a score of under 35 is considered low risk. Therefore a repeat prostate biopsy is warranted in this patient. PCA3 is now FDA-approved for the assessment of patients with a prior negative prostate biopsy. PCA3 is not affected by infection, prostate volume, or the use of 5-alpha reductase inhibitors. Aubin SM, Reid J, Sarno MJ, et al: PCA3 molecular urine test for predicting repeat prostate biopsy outcome in populations at risk: Validation in the placebo arm of the dutasteride REDUCE trial. J UROL 2010;184;1947-1952.

2013 - 12 A 71-year-old healthy, uncircumcised man has a 4 cm penile tumor and undergoes partial penectomy. Pathology reveals high grade squamous cell carcinoma invading the corpora cavernosum with negative surgical margins. After four weeks of antibiotic therapy, staging evaluation reveals bilateral bulky fixed, inguinal adenopathy and bilateral pelvic adenopathy. The next step is: A. pelvic lymph node biopsy. B. sentinel inguinal lymph node biopsy. C. XRT to inguinal nodes. D. bilateral pelvic and inguinal lymph node dissection. E. neoadjuvant cisplatin, ifosfamide, and paclitaxel.

E . This patient has a T2 (invasion into the corpus spongiosum or cavernosum) N3 (palpable fixed inguinal lymph nodes or nodal mas, either bilateral or unilateral) disease. In patients with unresectable primary tumors or bulky regional lymph node metastases, neoadjuvant treatment with a cisplatin-containing regimen is the most effective treatment modality and may allow curative resection. A phase 2 study using four courses of neoadjuvant paclitaxel, ifosfamide and cisplatin chemotherapy for TxN2-3 disease followed by bilateral inguinal lymph node dissections, and unilateral or bilateral pelvic lymph node dissections revealed excellent response with an objective response rate of 55% and complete pathologic response rate of 10%, toxicity was acceptable with no treatment-related deaths. This treatment is superior to single agent chemotherapy and has less toxicity than the previous multi-agent chemotherapeutic regimen of cisplatinum, bleomycin, and methotrexate. The optimal chemotherapy regimen however has yet to be determined. In this patient with bilateral bulky fixed nodes not responding to antibiotics, a needle biopsy of the lymph nodes could be considered for pathologic diagnosis. However, neither pelvic lymph node biopsy, sentinel inguinal lymph node biopsy, nor bilateral pelvic inguinal and inguinal lymph node dissection would be curative and would predispose the patient to non-healing surgical incision sites. Similarly, XRT to the inguinal nodes would not be curative for this extensive disease. Trabulsi EJ, Hoffman-Censits J: Chemotherapy for penile and urethral carcinoma. UROL CLIN N AM 2010;37:467-374. Pagliaro LC, Williams DL, Daliani D, et al: Neoadjuvant paclitaxel, ifosfamide, and cisplatin chemotherapy for metastatic penile cancer: A phase II study. J CLIN ONC 2010;28:3851-3857.

2013 - 149 A 76-year-old asymptomatic man has castrate-resistant prostate cancer. Bone scan is normal but CT scan of the pelvis demonstrates two new, enlarged pelvic lymph nodes. The next treatment is: A. sunitinib. B. zoledronic acid. C. denosumab. D. cabazitaxel. E. sipuleucel-T.

E . This patient has asymptomatic castrate-resistant prostate cancer. The potential next steps are docetaxel chemotherapy or immune therapy with sipuleucel-T. Both are FDA-approved in this clinical situation. Sipuleucel-T is an active cellular immunotherapy that is a type of therapeutic cancer vaccine. It consists of autologous peripheral blood mononuclear cells with antigen presenting cells that have been activated ex vivo with a recombinant fusion protein that consists of prostatic acid phosphatase that is fused to granulocyte-macrophage colony-stimulating factor (an immune-cell activator). In men with asymptomatic or minimally symptomatic castrate-resistant prostate cancer, a 4.1 month median overall survival benefit was demonstrated compared to placebo. This patient has asymptomatic castrate-resistant prostate cancer with soft-tissue disease. Sunitinib is a tyrosine kinase inhibitor and although FDA-approved for kidney cancer, it has not been approved by the FDA for men with prostate cancer. Zoledronic acid is an I.V. bisphosphonate indicated for patients with metastatic castrate-resistant prostate cancer with bony sites of disease. It has been demonstrated to decrease pain as well as skeletal-related events. No survival advantage has been demonstrated. Although it has been studied for men on long term androgen deprivation therapy, this patient has been on therapy for only six months. Denosumab is a RANK-ligand inhibitor and also decreases skeletal-related events and helps to prevent skeletal-related events associated with osteoporosis. Cabazitaxel is FDA-approved for patients with metastatic castrate-resistant prostate cancer who have failed docetaxel. This patient has not had any chemotherapy. Although not listed, docetaxel is FDA-approved agent for men with castrate-resistant prostate cancer and may be an appropriate next treatment. Kantoff PW, Higano CS, Shore, ND, et al: Sipuleucel-T immunotherapy for castration-resistant prostate cancer. NEJM 2010;363:411-422.

2012 - 131 A patient with a history of low-grade Ta superficial urothelial carcinoma of the bladder has three new bladder lesions 14 months after last resection. Histology reveals that two of the tumors are Ta while the third is T1. No muscularis propria is seen in the specimens. The next step is: A. surveillance cystoscopy every three months. B. intravesical mitomycin C therapy. C. intravesical BCG therapy. D. intravesical BCG plus interferon therapy. E. repeat transurethral resection.

E . This patient has not been adequately staged which makes the choice of an appropriate therapy difficult. According to the AUA Guidelines on superficial bladder cancer, in the absence of muscularis propria, data suggest that 20 to 40% of patients will have residual tumor and/or unrecognized muscle invasive disease. Therefore, repeat resection is considered a standard in this setting. All other therapeutic options would be inappropriate at this point because the patient has not been completely staged. Hall MG, Chang SS, Dalbagni G, et al: Guideline for the management of nonmuscle invasive bladder cancer: (Stages Ta,T1, and Tis): 2007 update. MANAGEMENT OF NONMUSCLE INVASIVE BLADDER CANCER GUIDELINE. American Urological Association Education and Research, Inc, 2007. <a href='http://www.auanet.org/content/guidelines-and-quality-care/clinical-guidelines.cfm?sub=bc' target='_new'><u>http://www.auanet.org/content/guidelines-and-quality-care/clinical-guidelines.cfm?sub=bc</u></a>

2014 - 5 A 21-year-old man receives a stem cell transplant with high-dose salvage chemotherapy for a Stage III NSGCT refractory to primary chemotherapy. After his salvage chemotherapy, his markers normalize and his CT scan is shown. The next step is: A. observation. B. PET scan. C. percutaneous biopsy. D. two additional cycles of chemotherapy. E. RPLND.

E . This patient has several residual masses after salvage chemotherapy. Given that he has received salvage chemotherapy and that his markers are normal, his best option is RPLND. The chance of viable disease is approximately 50%, teratoma 40%, and necrosis/fibrosis only 10%, thus, observation is a poor choice. A PET scan would not tell about teratoma which would need to be resected. A percutaneous biopsy could have sampling errors and would not obviate the need for additional treatment. Since the markers are normal, there is no indication at this time for additional chemotherapy especially in the salvage setting. Sheinfeld J, Bosl GJ: Surgery of testicular tumors, Wein AJ, Kavoussi LR, Novick AC, Partin AW, Peters CA (eds): CAMPBELL-WALSH UROLOGY, ed 10. Philadelphia, Elsevier Saunders, 2012, vol 1, chap 32, p 887.

2011 - 21 The prostate biopsy technique that samples the anterior prostate gland most effectively is the: A. 10-core biopsy. B. 12-core biopsy. C. saturation biopsy. D. transurethral biopsy. E. transperineal biopsy.

E . Traditional 10-core, 12-core, and even saturation biopsies utilize a transrectal approach, which limits access to the anterior prostate. Under regional or general anesthesia, the transperineal approach uses a template system to sample the prostate. This grid system is similar to the one used for brachytherapy. The advantage to the transperineal approach is its optimal access to the anterior and apical gland, particularly in high-risk patients with prior negative biopsies. The 10-core, 12-core, and saturation biopsy are most apt at sampling the peripheral zone. Ramey JR, Halpern EJ, Gomella LG: Ultrasonography and biopsy of the prostate, in Wein AJ, Kavoussi LR, Novick AC, Partin AW, Peters CA (eds): CAMPBELL'S UROLOGY, ed 9. Philadelphia, Saunders Elsevier, 2007, vol 3, chap 92, pp 2889-2892.


Kaugnay na mga set ng pag-aaral

ENVS Chapter 22, Apes Chapter 22, Chapter 22 Test

View Set

Ch 10: Acids and Bases and Equilibrium

View Set